personal statement tips lsat

  • LSAT Exam Info
  • What's Tested on the LSAT
  • What's a Good LSAT Score?
  • LSAT Raw Score Conversion
  • LSAT Test Dates
  • When to take the LSAT
  • LSAT Study Plans
  • LSAT Tips and Tricks
  • Applying to Law School
  • LSAT Pop Quiz
  • LSAT 20-Minute Workout
  • Free LSAT Practice Test
  • LSAT Question of the Day
  • LSAT Prep Courses

how to write a personal statement for law school

How to Write a Law School Personal Statement

Many prospective law students can feel overwhelmed when faced with the task of writing a law school personal statement, one of the most subjective pieces of your law school application .

A good personal statement is interesting to read without needing to rely on shock value. It should have a conversational tone; it’s not there to show how many big words you know, but rather to offer insight into your character.

TABLE OF CONTENTS:

Why Are Law School Personal Statements Important?

Tips for picking a law school personal statement topic, law school personal statement faqs.

  • How To Format Your Law School Personal Statement
  • How To Write a Great Law School Personal Statement
  • How To Edit Your Law School Personal Statement

Law School Personal Statement Example

What you should not do in a law school personal statement.

Law school personal statements are important because they can turn what would have otherwise been a certain rejection into an offer of admission.

They help admission committees get to know you in a way they couldn’t from other pieces of your law school application. And, hopefully, it also shows why a law degree is the next logical step for you.

When deciding what to write in your personal statement, do not make stuff up. Stick with what has really happened to you and how it affected you, and you will write a better personal statement than if you pretend.

Do Your Research

Read as many personal statements as you can. Discovering what has worked vs what doesn’t work is equally important.

Admission committees are very experienced at reading personal statements. They can quickly sniff out when something rings false.

Be Yourself

Admission committees utilize the personal statement portion of the application to learn about you and why you want to go to law school. Tell your story, and do not try to hide who you are.

Discuss Personal Stories with Friends and Family

Try getting feedback on your topic ideas from family or friends.  If they have been to law school, even better, but talking about your topic and learning how to articulate why you chose that topic for your personal statement will help you flesh out good ideas vs bad ideas. 

[ RELATED: LSAT Admissions Consulting ]

Try a Reflective Writing Exercise

You should get in the habit of being able to think deeply about how your interests and experiences relate to attending law school. Try writing a sentence or two for each of these prompts to get warmed up:

  • What you’ve done
  • Why you did it
  • What it meant to you
  • How it affected you

Then try writing a paragraph or more in response to these prompts:

  • Recount a time when you faced a challenge, setback, or failure. How did it affect you, and what did you learn from the experience?
  • Reflect on a time when you questioned or challenged a belief or idea. What prompted your thinking? What was the outcome?
  • Describe a problem you’ve solved or a problem you’d like to solve. Anything of personal importance, no matter the scale. Explain its significance to you and what steps you took or could be taken to identify a solution.

The personal statement is an integral part of the law school application, and it is important that you not only take it seriously but also try your best to have fun with it. Many questions may be circulating in your head that you feel need to be answered before you start to write your personal statement. Let’s look at frequently asked questions applicants have about writing their personal statements.

How long are LSAC personal statements?

Some law schools ask for only five hundred words, while others allow for up to four pages double-spaced. However, most schools ask for a two-to-three-page (double-spaced) essay.

Law schools have personal statement length requirements for two reasons: (1) to test your ability to follow directions (keeping to the length requirement) and (2) to evaluate how well you write.

What can I expect from the personal statement prompt? 

Most law school personal statement prompts are pretty vague and give the applicant a lot of room to interpret it as they see fit. 

Personal statement prompts are usually a general question and contain a page or word limit; exact requirements will vary from one school to the next. If you have questions you should get in touch with the admissions office via phone or email.

How personal should my personal statement be?

Incorporating emotion into your personal statement could make it more interesting and easier to read, but if you overdo it you can sound like you’re whining, begging, or trying to write a sob story—which can, in turn, be perceived as disingenuous.

Admissions committees want to see passion, but they also want to see who you are. It is extremely important to be honest. Law schools can see right through feigned emotions. Remember, they’ve probably read hundreds of thousands of these, and it’s very easy for them to detect when people aren’t being authentic. Allow yourself to be vulnerable, open, and clear—but make sure it comes from the heart.

A law school personal statement does not mean a mandatory hardship story. While getting through a rough life situation can be a great place from which to pull material for your personal statement, it is not even close to the only way to write a stellar piece, especially if it means overstating reality or making up emotional lessons that weren’t really present.

The biggest key to the law school personal statement is to be honest. The story you want to tell about how you’ve gotten where you are today doesn’t have to be exciting or on a grand scale or heartbreaking, it just needs to show something important about you.

“If you write about your childhood in your personal statement, you must find a way to tie it to your adulthood.”

Should you discuss your decision to attend law school in your personal statement?

This topic is contentious. If the rest of your application does not clearly indicate why you are applying—say, for example, you have no legal internship or student group experience, majored in a subject unrelated to law, and spent the past five years working in a biology lab—then you should at least touch on your reasons for pursuing a law degree in your personal statement.

However, if your application already demonstrates why you are applying to law school—whether through your college extracurricular activities, your work history, or your coursework—then you are probably safe to submit a personal statement that does not directly mention your decision to attend law school.

How to Format Your Law School Personal Statement

Law school personal statements should be:

  • Double-spaced, left-aligned
  • In 11 or 12-point Times New Roman font
  • Uses a single space after a period
  • Contain page numbers
  • Use one-inch margins on all sides
  • Your LSAC number
  • The words “Personal Statement”

Signatures and titles are not needed for law school personal statements.

How to Write a Great Law School Personal Statement

The trick to writing is getting that first word, sentence, or paragraph on the page; after that, everything can follow more easily. If you are having trouble starting your intro paragraph, start with the body of your essay. Saving the introduction or conclusion till the end is much easier for some people.

In each section of your essay, bring in references to who you are and how you will enhance the law school’s student body. Instead of just saying that you are diligent and compassionate, say that your experience training to run a marathon taught you the value of consistent hard work, and the time you spent volunteering with Habitat for Humanity showed you how important it is to empathize and help the under-resourced members of society. 

Find ways to make the things that you’ve done support your contention that you’ll bring something great to the law school.

Start with an Outline

Take your thoughts and organize them into an outline. Try to incorporate key attributes about yourself into your headings.

Remember that legal professionals place a high value on organization—a good personal statement is clearly organized and easy to follow—and since at least some of the people reading and evaluating your essay are legally trained, creating a good outline is crucial.

Personal Statement Intro Section

Starting a personal statement can be difficult and it may be easier for you to leave this section of your personal statement as the last thing you write. Use this section to introduce yourself, catch the attention of the reader, and set up your story.  

Tips for writing an introduction paragraph

  • Avoid catchy openings like “from a young age” or “I have always wanted to…”
  • Get to the point quickly
  • Start with something interesting, relevant, and specific
  • Think about what is inspiring you today
  • Avoid exclamation points to emphasize your excitement

Personal Statement Body Section

The body of your personal statement should focus on the details of your story. Each paragraph should expand on your points and begin with a topic sentence that expresses the main idea of the paragraph in which it occurs.

Ending sentences for body paragraphs should wrap up your points and help transition the reader to the next body paragraph or the concluding paragraph.

In the climax of your essay, use concrete language and zoom in on the moment of transition.

For example, if you talk about your financial stress, which was caused by foreclosure and you didn’t have an attorney, which led to you getting sick, don’t say that you “experienced hardship” or that it  “took a toll” on your health. What was the hardship? What was the toll? Specific, concrete details give transition moments their power.

[ TIP: Vary your sentence length to keep readers interested ]

Personal Statement Conclusion Section

The conclusion section of your personal statement should re-emphasize and summarize your main points.  It should be concise and leave the reader with a greater understanding of who you are and why law school is the next logical step for you in your education.  

If you are having trouble, consider taking a break and asking for help .  

Overcoming Writer’s Block

We all suffer from writer’s block sometimes, and it can be particularly brutal when the stakes are high … like when you are trying to get into law school.

One way to cut through the blockage is to make a stream-of-consciousness list of word associations. Start with a memory (e.g., first grade), a person (e.g., grandmother), or a place (e.g., the beach), and then just write down every word that comes to mind for the next two to three minutes. Do not worry if the words that come to mind are absurd (e.g., tuna, rabbits, dominoes)—they are coming from somewhere, and one of them just might trigger a memory that makes you think, “Ah! That could have a place in this essay.”

This advice may sound a little silly and unguided, but that is precisely the point. When you are experiencing writer’s block, that is a sign that you are too “in your head”—that is, your conscious mind. You need to hop over from your left brain to your right, which is less judgmental and more creative.

How to edit your law school personal statement

Do not underestimate how crucial editing is to writing a good personal statement. Editing is about far more than correcting your spelling and punctuation. A hastily edited personal statement could very well be the thing that makes the difference between “Congratulations!” and “We regret to inform you…”.

Inspect the Structure

Your first goal should be to make sure that your personal statement is well organized. Return to the outline that you wrote and shift things around if necessary. Make sure each topic sentence inspires you to keep reading.

Copyediting

Run the spell-check, of course, but also read through on your own, very carefully. If your typo is a correctly-spelled but inappropriately used word, it won’t set off the spell-check. Pay attention to your use of commas, semi-colons, and other punctuation marks; consult a resource on English language mechanics if you have any doubts about usage.

[ TIP: Make sure that you mention the correct law school in the essay ]

Verify Personal Statement Is About You

Avoid including too much about “the world” and/or too little about yourself.  Look for these items throughout your personal statement.

  • You have several sentences in a row describing life (or the universe, or society, or the world) in abstract terms.
  • You spend a full paragraph talking about something or someone else without reflecting on your topic from your perspective.
  • You get to the end of the personal statement and realize you do not know how what you have read reveals something significant about you as a person.
  • You spot very few uses of “I” in your personal statement.

If any of these describe your current draft, look for ways of introducing yourself more frequently in it.

Get Feedback From Others

Once you’ve fully completed editing, ask several people whose writing skills you trust to look over your essay and offer suggestions. Ask them if they came away with a clear and cohesive sense of you as an individual. 

Incorporate Feedback

When you’ve gotten feedback from others, incorporate suggestions you find valuable into your rewrites. Repeat this as necessary until you get an essay that you’ve proud of… or until your application is due, whichever comes first.

Note: To maintain the integrity and authenticity of this project, we have not edited the personal statements, though any identifying names and details have been changed or removed. Any grammatical errors that appear in the essays belong to the candidates and illustrate the importance of having someone (or multiple someones) proofread your work.

Personal Statement

I don’t imagine the process of coming out as gay is easy for anyone. I can still remember the first time the words came out of my mouth. The person I told, my best friend, waited expectantly for the big news I had promised her over the phone. My heart began to beat faster. My palms were sweating. A million thoughts raced through my head. Here was something integral to my identity, something so deep it had taken me years to uncover. And I was about to tell someone who could either accept it, or turn away from me.

Fortunately, the experience in my case was a positive one, overall. Without fail, my closest friends and family told me they loved me, and would continue to do so. There were, of course, some people who did not accept me, and that hurt in ways that I can’t begin to explain. But the ones who really mattered embraced me, and coming out to them was an affirming experience. I knew even more than I had before that I had a network of people around me who cared for me and supported me.

When I was in college, I became involved in activities that affirmed my identity further. I organized on campus for things like a gay student union and gender non-specific bathrooms, and the groups I worked with had various levels of success with these projects. But [my undergraduate university] is a largely queer-friendly school in [a large metropolitan city], and so the activities felt somewhat sheltered. After organizing with these campus groups for a while, I branched out and began volunteering for organizations in the larger city ….

I had always known that not everyone’s experience of coming out as gay was as positive as mine, but it was when I became involved with these organizations that I began to see just how cruel the world could be to LGBTQIA [lesbian, gay, bisexual, transgender, questioning, intersex, and asexual] people. I met thirteen-year-olds who had been abused and thrown into the street because they were gay. I met trans women who had been discriminated against for their identities by bosses and landlords. I met drag queens whose daily experience involved street harassment and the threat of bodily harm. For the first time in my life I was surrounded by people who were struggling every day to meet their basic needs like food and shelter because of their identities.

I also began to learn from people who were older than me, who had slept on the Chelsea Piers, and lived through the plague of HIV and AIDS. I learned about intersectionality, the varied forms that oppression can take and where they meet in an individual’s life. I learned of how mainstream organizations like HRC [Human Rights Campaign] and those involved in the fight for marriage equality often jettison the most vulnerable members of queer struggle in order to achieve what they consider the “greater good”—like the exclusion of transgender people from the Employment Non-Discrimination Act in the ‘90s. I learned about assimilation of gay people into mainstream society, and how it worked remarkably well for some while for others it would never, ever be an option.

In the end, it was these—the most vulnerable members of my community—that I found the most reason to fight for. People whose doctors won’t treat them because they’re HIV positive. Trans men and trans women without legal documentation who can’t find a lawyer that will take them on. Intersex prisoners who the prison industrial complex tries to squeeze into its limited boxes.

I honestly believe going to law school is the best way I can help these people. I have spent years writing and signing petitions, organizing LGBTQIA dance parties, protesting in the streets. Now, as I enter the phase of my life in which I am choosing a profession, I want it to be one that takes all I have learned and keeps it in the forefront of my mind. I want to stand up for the people in my community who have so few advocates.

A queer utopia—that is, a world in which the struggles I have learned of through my involvement in the LGBTQIA community no longer exist—is still a long way off. But I have seen good people filling in the gaps in the lives of those most strongly affected by inequality. I am committed to becoming one of those people, and I feel that this is the best way I can do it.

A story illustrating the reasons you want to go to law school is always going to be more effective than a generic essay that anyone could have written; remember the point of the law school personal statement is to show a law school something unique about yourself. 

Law School Personal Statement Don’ts

  • Avoid dramatic tales of romance
  • Curse often
  • Avoid absolute statements which tend to sound more unreasoned than reasoned— law school is all about reason
  • Do not submit an essay version of your resume
  • Do not use the same personal statement for every school

[ NEXT: What not to do in a law school personal statement ]

You might also like

lsat-law-school-admissions-test-raw-score-conversion-conversions

Call 1-800-KAP-TEST or email [email protected]

Prep for an Exam

MCAT Test Prep

LSAT Test Prep

GRE Test Prep

GMAT Test Prep

SAT Test Prep

ACT Test Prep

DAT Test Prep

NCLEX Test Prep

USMLE Test Prep

Courses by Location

NCLEX Locations

GRE Locations

SAT Locations

LSAT Locations

MCAT Locations

GMAT Locations

Useful Links

Kaplan Test Prep Contact Us Partner Solutions Work for Kaplan Terms and Conditions Privacy Policy CA Privacy Policy Trademark Directory

Law School Personal Statements What Not To Do

LSATMax Logo

Law School Personal Statement

By Mehran Ebadolahi Mehran Ebadolahi -->

person writing law school personal statement

While an LSAT score is important, there are other aspects of your application that are fundamental for your admission to law school. Taking a moment to look a bit beyond LSAT prep, it's time to discuss your upcoming personal statement?without it, you'll struggle to get into your first choice no matter how well you do on the test.

Right now, most of your focus is probably on your admissions test, and your applications may seem miles away to you right now. Especially if you're in the studying process, your personal statement is probably the last thing on your mind. But your statement is part of a large matrix of what a school takes into account when admitting students, and your applications will creep up on you much sooner than you think.

Many of the top law schools have said that right after your LSAT score and your GPA, they look at your personal statement. Chances are there will be many students that apply to your target law schools with similar GPAs and LSAT scores. Because of this, one of the only ways you can make yourself stand out is through your personal statement.

The Importance of Personal Statements

So, your GPA is impressive and your LSAT scores make you eligible to apply for some of the top law schools in the country. Congratulations, the hardest part is over!

Now, it's time to pay attention to the only other thing you can use to truly differentiate yourself from all the other top academic applicants at these universities.

The truth is that no matter how smart you are and how high your test scores are, you're diving into a peer group that's just like you. If you're applying to top law schools, there's a good chance many of the other applicants have a similar level of academic excellence as you. They've done extracurriculars, they have great grades, and they've done well on their LSAT.

All you have to differentiate yourself is your personal statement. It's the one piece of your application that no one else can mimic or copy. When you're on even playing ground with others academically, it's time to stand out from the pack with a fantastic personal statement. That's why it's not just important to write a solid, convincing statement, but to use your statement to stand out from the crowd.

There's no checklist or instructions for a personal statement. You can't treat it like a regular academic assignment. No amount of knowledge will help you excel above the other applicants. Your impact will come from writing something that's compelling and persuasive.

Think of it like making a case for yourself. With your personal statement, you're answering the question:

Why should we choose you?

It's a deep and difficult question with no simple answer. But, it's also your best chance to dig deep and convince the admissions officers that you have what it takes to be successful at their law school.

How to Write an Impactful Personal

Writing the perfect personal statement is a delicate balance. You must show what makes you uniquely suited to the challenges of law school, as well as showing how you fit the specific school's culture and atmosphere. There are as many ways to succeed in this effort as there are to fail, so it's important to consider your approach thoughtfully.

Consider your Target

One student recently took a leap of faith, and wrote about his status as an extinct mammal because of his natural red hair. If you aren't aware, red hair is a recessive trait that is slowly being weeded out by natural selection. People with red hair are actually becoming less and less numerous with each passing generation. This student wrote about how he should be considered a minority (though he is a Caucasian male) because redheads will soon be obsolete.

Many of the law schools didn't seem to find his statement amusing. However, his top-pick school sent him his admissions packet along with a copy of his personal statement with a sticky note attached to it.

The sticky note stated how much they enjoyed his statement because it was well written and thoroughly showed them his personality and wit. It goes to show that your statement doesn't have to please everyone, just your top picks.

Be Creative

Another example of the importance of personal statements comes directly from the mouth of an admissions officer from Columbia University. When asked about personal statements, the admissions officer in question stated that each year the admissions counselors at Columbia compete with each other to find the best personal statement.

This year, the statement belonged to a man who wrote about his workplace. Apparently, every Friday at this applicant's office, there was an obstacle course race that each office worker participated in. It consisted of each participant sitting on an office chair and pulling themselves through the office.

This person was the all-time reigning champion of the obstacle course race, and when this fact was brought to his attention, he realized he had stagnated and needed to make a change in his life. Therefore, he decided to veer his life path towards law school.

The admissions counselors all loved the statement and chose it as the best of that year. Consequently, that person was admitted to Columbia, and all because of how well written his personal statement was. Can you imagine getting into your dream school based on a story about rolling your office chair around obstacles?

Focus on the Long Term

Anecdotes aside, you need to remember that there is still a path ahead of you after the LSAT. Even after you finish, there are really important things you need to focus on.

So, it's prudent that once in a while, in your LSAT prep off-time, you should begin to ruminate on topics that might work for your personal statement. Thinking about it early will allow your brain to start subconsciously reflecting on your topic.

Keep in mind that your LSAT isn't your final goal. The true goal of this painful process is to get into law school, then to graduate and find work in the field of law. As much as the LSATs are a vital part of that, you can't lose sight of the overall vision.

LSATs are a major part of your law school application. However, once you're into law school, your LSAT score won't make much of an impact on your future career. The same is true of your personal statement, although it can help to reveal more about you as a person. In the end, your personal characteristics and driving motivation will impact your future far more than LSAT scores.

Take time to do well on your LSATs. But, don't neglect the other parts of your applications, like the personal statement, that might have an equally meaningful impact on your admission.

Now, it's time to learn more about personal statements and how to write one for yourself.

Introduction to Personal Statements

No law school application is complete without a personal statement. The application is typically two pages in length, and your personal statement is likely the most important qualitative (non-number-based) element to your application.

It shouldn't be treated as a second thought by any means. Frequently, it's pointed out that the personal statement is an admissions counselors' entry point into your application, and you know what they say about first impressions!

Be Yourself or Sell Yourself

There are two schools of thought with the personal statement. One school believes that the personal statement is where the true you should shine; step away from the numbers you've acquired and your academic pedigree and really let the admissions officers know what makes you tick, what makes you, you.

The other school of thought believes that the personal statement is not a place for flowery stories about moments that have changed your life, and rather it is another opportunity for you to highlight your academic achievements.

What you should do, if you have the opportunity, is do some research on the admissions department of your target school. A great way to do that is to set up a meeting with them. That way, you can talk to them in person, and maybe even get a chance to tour the campus and the library (a place you will be spending most of your time if you go to that school).

When you are in the meeting you can get the info you want straight from the horse's mouth. Then, you can write your statement appropriately. Online research and forums can also help you get specific information on a particular law school.

What to Talk About in Your Statement

As for the subject matter, we've established that your personal statement should be about you, of course, but what about you exactly? The answer is to write about something personal, relevant, and as unique to you as possible. This could be a quality you possess, an experience that helped define you, or motivation that drives you.

Whatever you choose, the key is–you guessed it–to make it personal. It doesn't have to be an event that has likely never happened to anyone else, a feeling no other person has ever felt, or a trait no other person has possessed; but keeping authenticity and honesty in your essay will drive home the personal element.

Consider your personal statement to be the creative part of your application. This is the element that goes beyond quantitative credentials and lets you win over the admissions officials on a personal level.

Brainstorming Your Personal Statement

There's no perfect answer to the subject matter you should write about. If you need some help coming up with a topic, follow a process like this:

  • Write as many topic ideas as you can think of, ignoring whether they're good or bad. Make sure this initial list is extensive and thorough. It should include ideas in a broad range of topics, from a personal experience that influenced your life to how you've overcome challenges or circumstances that contributed to your character. Don't be shy in this phase, and start it early: taking your time will help your brain come up with more ideas.
  • Read through your list of ideas and cross out any that don't fit in well with the initial criteria. Focus on getting rid of ideas that aren't about you directly, aren't very impactful, don't reveal part of your character, or are circumstances you're not very comfortable talking about in-depth.
  • Once you've narrowed down your list, look at any remaining options and create pros and cons lists for the remaining options. If the first cut didn't narrow it down enough, do another cut before making and pros and cons. Make your decision based on the pros and cons of each idea, choosing the one that makes the strongest case for your admission.

If you don't feel confident in your topic choice, pick another topic. It's okay if it feels a little out of your comfort zone, but you don't want to pick a topic that makes you feel so uncomfortable that you can't write about it well. You need to be able to write a thorough, clear, and compelling statement about whatever topic you choose, so choose something you'll be able to expand on.

The Elements of a Great Personal Statement

No matter what style you're going for or what you're writing about, each law school personal statement has a few common elements. These are fairly consistent throughout all statements. Getting the structure correct and submitting the right elements will improve your chances of gaining entrance into a top law school.

Here are some things to consider when creating the perfect personal statement for your law school application:

Introduce Yourself

It's essential to begin your statement with a great opening. The University of Chicago Law School recommends against beginning with a quote, as many choose to do. Starting with a quote, mantra, or vague idea doesn't immediately present you.

This is, after all, a personal statement: lead with yourself. Come back to fundamentals; your personal statement is about you .

The admissions officials are examining your personal application, not a general application to judge for adequacy (that's the job of the LSAT). Begin honestly, and remember that the audience you're writing for has read innumerable personal statements (even just this year) and they'll smell lies from a mile away. Don't waste your own time or their time trying too hard to be clever.

Try to start on your best foot. Our advice is to write the introduction last. Write the meat of your personal statement first, then loop back and write your introduction. That way, you'll be able to come up with a better opening statement that ties in well with what you're talking about in general.

Tell Your Story

The bulk of your focus should be on the real substance of your personal statement. Make sure that you're following a logical narrative structure. Personal statements should be more of a story than a persuasive essay. If you're sharing an interesting personal experience or life event, you'll be best served if you think about:

If any of these elements aren't clear in your personal statement, you'll have a bit of difficulty formulating a compelling narrative to convince an admissions officer of your merit. Review your statement and make sure you're touching on everything, and how each part relates to you.

Taking a closer look at the earlier example from the chair-racing gentleman, we see all 5 of these elements come into play: The time is given, the location of the office is mentioned, the chair racer identifies himself in the story, the action of the chair races and being declared the all-time champion is discussed, and the outcome of the decision to pursue law school is revealed.

This is just a guideline for structuring the body of your personal statement. Because it's generally a story about you, it's in your best interest to make sure you're doing your story justice.

Make Your Point

The conclusion of your statement should be where you reiterate the message of your personal statement and answer the question of what you're a good candidate for admission. It's all about wrapping up the whole statement into a neat and tidy ending that creates a sense of closure and calls the reader into action.

In this case, the action you want is acceptance into the school. Don't be too outright?after all, you're applying to law school, but look to touch on points of empathy and connection. Make it hard for the admissions counselor to reject you, make it easy for them to say yes.

Often, your conclusion will look a bit like your introduction, because you'll likely talk about similar themes in both. However, the goal of the conclusion is to leave the reader with a satisfying sense of closure and to lead them to appreciate the qualities you've emphasized in the statement.

General Do's and Don'ts

If you have to narrow it down to one solid list of things you definitely should be doing and things you definitely shouldn't be doing, here's what you would end up with:

  • Be concise, organized, and candid
  • Proofread, proofread, and then proofread again
  • Be yourself
  • Keep your personal statement about you
  • Be honest and authentic
  • Connect your story to the profession of law
  • Outline before you write
  • Regurgitate your resume and qualifications
  • Overlook typos or submit anything but the final version
  • Name drop (it's okay to touch on an important figure in your life, though)
  • Attempt to use legal terms or phrases you may be using incorrectly
  • Cover your entire life story

If you need something more in-depth, Nova.edu put together a guide to creating personal statements , and BU Law has some great examples of statements that cut through the noise.

Personal Statement Tips

Besides the structure and tone of the statement, what else can you keep in mind to make yours better? There are as many opinions as there are law school admissions officers, unfortunately. But, you can still glean some useful tips from the chaos of opinions:

No Gimmicks

Skip the gimmicks. Gimmicky personal statements scream ?I don't have anything original to say' more than they amuse. If you can think of it in under a minute, they've likely read it.

What they haven't read is a concise, creative, and thorough presentation of why you're unique, interesting, and valuable for their law school.

Take Feedback and Proofread the Statement

Don't rely only on your own judgment for your personal statement, have a trusted friend, colleague, or individual read your essay and give constructive, blunt feedback.

Your judgment is the ultimate decider, as it's your application and your statement was written about you, but you should always seek feedback from people you trust. Look for trusted academic advisors, mentors, or even friends and family who could offer a fresh perspective. Ask how compelling it is, how well written it is, and ask if they would make changes. Accept and apply the feedback that you think makes sense with your vision.

Beyond simple feedback, have a fresh set of eyes proofread your essay. It can't be said enough: proofread, proofread, proofread! Your personal statement is your introductory argument as to why you should be admitted to your chosen law school. If your essay contains errors, it's less likely an admissions officer will be inclined to believe you have the skills and attention to detail required for your legal studies.

Avoid Templates

Personal statements should be just that: personal. If you look up a template and essentially fill in the blanks, you're not giving the admissions officer a true look into your personality, your best qualities, and the unique contributions you can make.

Templates bring up a lot of problems. A little food for thought:

  • Did that example statement help the writer gain entry to law school?
  • Will a school admissions officer recognize the template format and stiff writing style?
  • Why would you trust your chance of admission to someone or some organization with no skin in the game?
  • Is using a template considered plagiarism?
  • How do you know if a template is even any good?

Just like writing your resume or CV, templates can only get you so far. You can look at the basic structure of a template for reference, but that's about all you should use them for. In fact, we recommend looking at successful personal statements, but be original!

Take the time to do this right, even if it means going around to different academic influences and people you trust to help you write, re-write, edit, proofread and polish your final personal statement. It will be worth it once you get that acceptance letter!

Wrapping Up

The perfect personal statement is the one that perfectly and truly states you. If you bring your personality, your voice, and your story with honesty and authenticity, you'll have the start of a winning essay.

There's the reason they call it a personal statement: it should be personal. Since admissions counselors can't get to know all of their applicants, they must use this to evaluate you. While it's difficult, if you can show off your personality and uniqueness, you've got a great chance for success.

#1 LSAT Course

Get the #1-Rated LSAT Prep for free.

See your chances of getting into top law schools., enter your email to see the full report.

LSData

The Ultimate Guide to Writing an Outstanding Law School Personal Statement

Dazzle admissions with your legally awesome personal story, introduction.

Let's face it: you've spent countless hours studying and acing the LSAT, and now it's time for the pièce de résistance – the law school personal statement. This is your golden opportunity to showcase your personality, and put your best legal foot forward. But don't worry, this guide has got you covered. In no time, you'll be writing a personal statement that could put John Grisham's early drafts to shame.

If you're ready to convince law school admissions committees that you're the next Ruth Bader Ginsburg or Thurgood Marshall, then buckle up and get ready for a wild ride through the world of crafting the ultimate law school personal statement.

1. Know Your Audience: The Admissions Committee

First and foremost, remember that you're writing for the admissions committee. These are the gatekeepers of your future legal career, and they've read more personal statements than there are citations in a Supreme Court decision. To avoid becoming a legal footnote in their memory, keep the following in mind:

  • Be professional, but also relatable. You don't want to sound like a robot that's been programmed to spout legalese.
  • Avoid clichés like "I want to make a difference" or "I've always wanted to be a lawyer." Unless, of course, you've been dreaming of billable hours since you were in diapers.
  • Consider what makes you unique. Remember, this is your chance to stand out among a sea of applicants with equally impressive academic records and LSAT scores.

2. Choosing Your Topic: Make It Personal and Memorable

When it comes to choosing a topic for your personal statement, think of it as an episode of Law & Order: Your Life Edition. It's your moment to shine, so pick a story that showcases your passion, resilience, or commitment to justice. Consider these tips:

  • Use an anecdote. Admissions committees love a good story, especially one that shows your problem-solving skills or ability to navigate tricky situations. Just be sure not to end up on the wrong side of the law!
  • Reflect on a transformative experience. If you've had a life-changing event that led you to pursue law, share it! Just remember to keep it PG-rated.
  • Discuss a personal challenge you've overcome. Nothing says "I'm ready for law school" like demonstrating your resilience in the face of adversity.

3. Structure and Organization: Your Legal Blueprint

Now that you've chosen your topic, it's time to draft your personal statement. Like a well-organized legal brief, your statement should have a clear beginning, middle, and end. Consider the following tips for structuring your masterpiece:

  • Begin with a strong opening. Start with a hook that will capture the reader's attention and make them want to keep reading. Think of it as your own personal Miranda warning: "You have the right to remain captivated."
  • Develop your story in the body. This is where you'll expand on your anecdote or experience, and explain how it has shaped your desire to pursue a legal career. Remember to be concise and avoid meandering – this isn't a filibuster.
  • End with a powerful conclusion. Tie everything together and reiterate why you're the ideal candidate for law school. Just like a closing argument, leave the admissions committee convinced that you're the right choice.

4. Style and Tone: Finding Your Inner Legal Wordsmith

When it comes to your personal statement, you want to strike the perfect balance between professional and engaging. After all, no one wants to read a 500-word legal treatise on why you should be admitted to law school. To achieve this delicate balance, follow these style and tone guidelines:

  • Write in the first person. This is your personal statement, so own it! Using "I" allows you to convey your unique perspective and voice.
  • Keep it conversational, yet polished. Write as if you were speaking to a respected mentor or professor. Avoid slang, but don't be afraid to inject a bit of your personality into your writing.
  • Employ dry humor sparingly. A little wit can make your statement more enjoyable to read, but remember that humor is subjective. It's best to err on the side of caution, lest you inadvertently offend the admissions committee.
  • Be precise and concise. Legal writing is known for its clarity and brevity, so practice these skills in your personal statement. Aim to keep it between 500 and 700 words, as brevity is the soul of wit (and law school applications).

5. Revision: The Art of Legal Editing

It's been said that writing is rewriting, and this is particularly true for your personal statement. Once you've drafted your masterpiece, it's time to don your editor's hat and polish it to perfection. Follow these tips for a meticulous revision:

  • Take a break before revising. Give yourself some distance from your statement before diving into revisions. This will help you approach it with fresh eyes and a clear mind.
  • Read your statement out loud. This technique can help you catch awkward phrasing, run-on sentences, and other errors that might not be apparent when reading silently.
  • Seek feedback from others. Share your statement with trusted friends, family members, or mentors who can provide constructive criticism. Just remember, opinions are like law school casebooks – everyone's got one, but you don't have to take them all to heart.
  • Edit ruthlessly. Don't be afraid to cut, rewrite, or reorganize your statement. Your goal is to make your writing as strong and effective as possible, even if it means sacrificing a clever turn of phrase or an endearing anecdote.

6. Proofread: The Final Verdict

Before submitting your personal statement, it's crucial to proofread it thoroughly. Even the most compelling story can be marred by typos, grammatical errors, or other mistakes. Follow these proofreading tips to ensure your statement is error-free:

  • Use spell check, but don't rely on it entirely. Some errors, like homophones or subject-verb agreement issues, may slip past your computer's watchful eye.
  • Print your statement and read it on paper. This can help you spot errors that you might have missed on-screen.
  • Enlist a second pair of eyes. Sometimes, a fresh perspective can catch mistakes that you've become blind to after multiple revisions.

Crafting an outstanding law school personal statement may seem daunting, but with the right approach and a healthy dose of perseverance, you can create a compelling and memorable statement that will impress even the most discerning admissions committee. So go forth and conquer, future legal eagles! And remember, as you embark on your law school journey, may the precedent be ever in your favor.

personal statement tips lsat

Tech-focused creator of LSD.Law. I built LSD while applying to law school. I saw unequal access to knowledge and built LSD to level the playing field and help applicants make thoughtful, well-informed decisions in the application process.

  • Data download

Help us make LSD better!

A Sample Structure for Your Law School Personal Statement

Nathan Fox

Applications open in early September at most law schools, and forward-looking students have started asking questions about their personal statements. “Where the hell do I get started?” is by far the most common of these.

Last Wednesday, I guest-taught the Admissions Hour with LSAT Demon tutor (and soon-to-be Yale 1L) Carl Lasker. In that session, we brought back our “Personal Statement Woodchipper,” wherein we read one brave applicant’s draft personal statement. And, well, we shredded it. Like always.

The biggest problem with this one, like so many others, was that it left us with no real picture of who the person is or what they do. There was too much cinematic scene-setting, too much industry jargon, and too much Tarantino-style timeline-skipping.

There was not enough of the actual applicant we were supposed to be learning about. In short, the personal statement wasn’t personal.

We each have a dozen different stories, any of which could make the foundation of a strong personal statement. So it’s less important that we pick the perfect topic, and far more important that we get our bad first draft out of the way ASAP. Until we have a draft, we can’t start cutting the worst parts and teasing out more of the good stuff. The purpose of this lesson is to help you get started.

This isn’t the only way to get started—it’s just one way. If you’re stuck, this might be a good way to get the words flowing.

The basic formula: I am. I did. I do. I will.

Ben and I talk endlessly about showing, rather than telling, any time a personal statement comes up on the  Thinking LSAT Podcast . The point is to demonstrate your strengths and achievements via facts, rather than forcing conclusions down the reader’s throat. Fact-driven writing is far more powerful than conclusion-driven writing. So the bulk of your statement needs to consist of sentences that follow the basic formula of “I did X.” We want your statement to be stuffed with sentences that feature you as the star of the show—don’t be shy about using the word “I” as the subject of your sentences—with active verbs. Here are some examples:

I wrote. I managed. I researched. I reorganized. I developed. I created. In short, I killed it.

Generally, you should avoid passive construction using forms of the verb “to be.” Steer clear of be, am, is, are, was, were, being, and been. When you see these verbs, replace them with something active.

Why, then, do I suggest starting with “I am”?

As Carl and I discovered in the Admissions Hour last Wednesday, the emphasis on active verbs can sometimes fail to give readers the necessary footing. We dive right into an action scene, without any background. So in this formulation of “how to write a personal statement,” it’s okay to start with a very brief statement of who you are, to help us understand the action to follow. Then skip back in time, just once, to discuss some background. Then progress into the modern day. Finally, if necessary, talk about the future. We’ll start in the present.

Ben Olson doesn’t let me use italics any more, but if I were allowed to use them I would italicize the “very” in “keep this section very brief.” I’m talking one or two sentences. See how it looks as a short, standalone paragraph. Using myself as an example:

I am the co-founder of LSATDemon.com, an LSAT preparation program with students and teachers from around the world. I’m also the co-host of the Thinking LSAT Podcast, which published its 296th episode last week.

This tells the reader who I am today, priming them for the story arc I’m planning to take them on. Don’t do more than a sentence or two of this, because it’s telling rather than showing. But the reader now has a picture of who I am in the world, which will help them comprehend the following sections.

Immediately, we transition back in time to provide some background. We’ll only do one such shift in the timeline—generally, chronological stories are much easier to digest. Readability is key, so we’ll only go forward, never backward, from here.

After graduating from Babson College in 2006, I began moonlighting as a GMAT teacher. My employer at the time needed an LSAT teacher, so I started teaching that as well. In 2008, in the summer between my 1L and 2L years of law school, I started Fox LSAT. I rented the back room of a Mission District cafe for my first class, which struggled to enroll a dozen students. But strong Yelp reviews and word of mouth filled subsequent classes.

From here, I’d continue to progress through my career. I’d talk about books I wrote, my in-person classes, the podcast, and founding LSAT Demon. This section can be anywhere from one paragraph to a page, depending on how much history is relevant.

Law schools are keenly interested in the person you are today. If they admit you, that’s the you who’s actually coming to class—not the you from five years ago. So we’ll shift to the present tense here to show the reader the strong, positive, winning applicant they’re looking for.

I now teach LSAT classes three days a week, and I love teaching now more than ever. I am especially proud of our free resources, including the Thinking LSAT Podcast, which have reached tens of thousands of students. I hire and manage a staff of two dozen freelance teachers, writers, and editors. Together, we offer multiple live classes seven days a week.

Here I could write about favorite students, or interactions I have with the teachers I mentor, or the laughs I have recording each week with Ben. You don’t have to choose the one perfect story, and each anecdote doesn’t have to be wildly impressive. Just demonstrate that you are capable, reliable, creative, thoughtful, resourceful—in other words, someone with their shit together.

Many excellent personal statements—especially for anyone who already works in anything even tangentially related to business or law—can end right here. You’ve shown who you are, what you’ve done, and what you currently do. Your reader is in the business of selling law school. They can put the pieces together between what you’re doing now and the opportunities they believe their law school can offer you. They already know you’re applying to law school, so don’t waste time with “I am applying for matriculation at your fine law school blah blah blah.”

But if you’ve been a musician for 15 years, or you studied chemistry in college, or your history-to-present doesn’t exactly scream law school, you may want to add a fourth, extremely brief section:

If it’s obvious that you’re changing career paths completely by applying to law school, try closing with a very brief statement about what you plan to do. One or two sentences is plenty, as you haven’t actually done any of this stuff yet—and it’s all just kind of BS until you actually do it. But you want your story to make sense (and hopefully it does, in fact, make sense), so go ahead and tell them what you’re trying to do.

I hope to gain experience in trade secret and copyright law. Eventually, I might explore a legal practice in the licensing of digital educational materials.

Boom, that’s it. Shitty first draft of personal statement complete.

I am. I did. I do. (And optionally, I will.)

That’s one or two sentences to introduce yourself. Then a paragraph to three-quarters of a page of what you did, leading up to three-quarters of a page of what you do now. End it there, or, if necessary, add one or two sentences of what you hope to do.

This isn’t by any means the only way to write a personal statement. But it’s at least a framework for getting something on the page that isn’t a complete mess. From here, if you find the glimmer of something you like, it’s all about the rewrites and editing.

LSAT Lessons

Get LSAT strategies in your inbox.

© 2024 LSAT Demon, LLC Terms of Service

personal statement tips lsat

In my time as a Pre-Law advisor and admissions coach, I’ve had the pleasure of working with some truly outstanding candidates. These are the folks who, before finishing college, have written extensively, published journal articles, or have completed post graduate fellowships like the Fulbright or Marshall Scholars programs. Others have gone off to have great careers in industry, or the military, before heading off to get their J.D. Regardless of who they were, or what they’d achieved, the process of writing a meaningful law school personal statement always proves to be a tricky and demanding task.

The good news is that you don’t need to be a Rhodes Scholar to write a compelling personal statement; nor do you need to talk about that time you cured cancer, or saved a boatload of kittens. That being said, it does mean that with adequate time, persistence, and planning, anyone is capable of writing the kind of personal statement that makes a difference. Before we jump into some helpful tips and tricks, let’s first understand what a personal statement is, and isn’t...

A lot of you, if you haven’t already, will be tempted to look at the examples of ‘winning’ statements that are out there. There are numerous books on how to write a personal statement (PS), many of which promise to guide the reader through samples of the ‘best of the best.’ If you’ve already looked at any of those books, do your best to forget them. If you haven’t read them yet, don’t.

It’s not that the essays in those books aren’t great, they are, but therein lies the problem. For someone who is embarking on writing their PS, it is easy to be overwhelmed with the 50 odd examples in a ‘How to’ book. Firstly, remember that these essays don’t all come from a single admissions cycle. Instead, they were carefully curated from several application cycles, often from a variety of top tier schools. Secondly, it is easy to read just one of those books and feel like your life has been so unremarkable, that you could never write a meaningful personal statement.

While such a conclusion is understandable, the words of well-known paper salesman, martial artist and beet farmer, Dwight Schrute, provide a more accurate assessment of it:

The beauty of the PS is not that it requires you to write the most incredible narrative ever conceived, nor do you need a world changing achievement. Instead, it needs to do what it says on the bottle - it needs to be personal. Remember, most schools do not interview candidates, and those that do, tend to do so only after an application has been submitted. The PS is intended to give the admissions committee an insight into who you are, what makes you tick, and to understand why law school makes sense for you. Now, this isn’t to be interpreted as a formula, but rather to drive home the point that you can engagingly convey who you are, without the need to rely on ‘I want to save the world’ or ‘study abroad’ tropes. It also means that an authentic personal story is all that’s needed, and you don’t need to worry about having some epic tale of achievement. Now, you might have a story that is, in fact, pretty incredible. Good for you, but for many applicants, they fear their self-perceived mediocrity. Fear not my friends, help is at hand!

Reflect, before you write

Before typing a single word, the most important thing to do is to unpack your own story. If you haven’t taken the time to really reflect on who you are, and what made you that way, you aren’t going to be able to write a truly personal, personal statement. This doesn’t mean you need to sit around meditating on the meaning of justice and your place in the world, but it does mean that you need to carefully reflect on your own journey. My suggestion is simple, but feel free to adjust it to suit yourself:

  • Find yourself a quiet place, away from your phone, laptop, or smart devices. Try to choose a place where you are unlikely to have a chance meeting with friends. Avoiding distraction is the name of the game.
  • Spend 30-45 minutes, once a week, reflecting on who you are. What motivates you? Who has influenced you or impacted your life? In short, why are you the way that you are?
  • Using a journal (as opposed to a laptop or the notes app on your phone – too many opportunities for distraction), make notes on those things mentioned in b (above).

As you undertake this task, people, situations, stories etc. will spring to mind – write these all down and, over time, you will build a database of ideas for a PS. This is likely to take some time, so don’t rush, and start this process well in advance of your application cycle. It is normal for a good PS to take at least 8-10 drafts, so be patient and start early. This is not something that you can write in a single weekend.

Once you have got 2-3 solid concepts, start to consider which might best serve the goal of a PS. Sketch out each concept so that it has some meat to it. Which one best conveys your story? Which allows the reader to get the truest sense of who you are? What details are important? Can this story be told in way the engages the reader? Does it leave the reader with a 1-2 sentence take away that moves you closer to being someone they want to admit? Careful planning will make the process of writing much easier in the long term, so resist the urge to write a draft until those concepts have been fleshed out, at least a little.

It is worth mentioning the importance of PS prompts. Many schools will offer a vague prompt as to what to address, others will give very specific guidance e.g. “Why do you want to attend this law school?” Where there is a prompt, address it clearly, but whatever the case, schools really do want your PS to tell them about you. Remember, it will serve, in most cases, as a replacement for an initial interview and the classic interview question, ‘Tell us a little about yourself.’ It will also be viewed as an example of your skill as a writer, much more than say, the essay you wrote as part of the LSAT.

Of course, writing a personal statement is something that needs to be crafted to suit the individual and what works for one person, might not work as well for another. But, if you spend the necessary time at the front end, conceptualizing what your PS will address, the actual writing will be not only be easier, but it will take a lot less time.

In addition to careful and thoughtful planning, the PS requires applicants to follow a few simple, but clear rules.

Quotes and metaphors

Believe me when I say, I love a good metaphor. I also love a good quote, so much so that I have two especially meaningful ones hanging on the walls of my office, but that’s where they belong. When writing your PS, always remember the goal of the essay. Throw off any pretense of trying to show off your vocabulary, and focus on writing in a way that really conveys who you are, and your ‘why?’ Write with sincerity and from your own, personal perspective.

The problem with using quotes is that you are using the genius or wisdom of someone other than yourself, which isn’t helpful in an essay with very limited space and a clear purpose. It can also make you sound like you’re trying a little too hard, even a little melodramatic – which is to be avoided: clichés kill a PS. The bottom line is that you need every line of your PS to have a purpose. Quotes and forced metaphors take up valuable space and achieve very little, so give them a wide berth.

Tell your story, not someone else’s

Your grandma might have been the most intelligent, badass woman of the 20th century. Maybe your uncle has some epic tale of overcoming adversity?

These people were probably as awesome as you say. Maybe you even undersold them, but how does telling their story, help your application?

Their stories may be amazing and inspiring, but as a law school, I don’t want to know about them; they aren’t applying to law school - you are. The admissions team wants to get a glimpse of the person applying, and telling the story of another person, no matter how interesting, fails to address the personal nature of a personal statement. If you are going to be referring to something that happened to somebody else, (which is risky) be sure to tell it from your perspective.

Humor and informality

Who doesn’t love a good laugh? I know I do, but the thing about humor is that for every person who loves Adam Sandler or Kevin Hart, there will be a group of people who either don’t find them that funny, or who straight up do not like them at all. Remember, even Dave Chappelle has tried out new material in a comedy club that just didn’t land. You are not Dave Chappelle, so don’t try to be – especially in your PS.

Humor can work in a personal statement, but it is a higher risk play and if it doesn’t land, it can come across as offensive, or immature. If that happens, you’ve made the admissions committee’s job a lot easier (and not in a good way!). If in doubt, always err on the side of authenticity over laughs and avoid the risk of an ill-conceived joke.

Don’t write an opinion piece

Opinion pieces have a place, just not in your PS. The simple reason being that with so little space, writing an opinion essay doesn’t help the reader really get to know you – which is the ultimate goal of the PS. It can also backfire horribly. Perhaps you wrote an essay on a particularly controversial issue? Worse still, perhaps you wrote the ‘I want to be a lawyer so I can change the world, and I know I can’ essay? This is actually much more common than you might think, and it needs to be avoided for two main reasons. Firstly, with so little space, you can’t write that essay and still achieve all that a PS is supposed to. Secondly, writing this essay can make you look incredibly naïve about the practice of law, which also suggests immaturity. That combination may not be fatal to your application, but it will give a school pause, regardless of your LSAT or GPA.

Put your best foot forward

While we all have endured different things, some of us have especially hard tales to tell. Can you use those stories in a personal statement? Yes, but it is all about the narrative you choose to tell. Is it a tale of woe, or of overcoming adversity? With that in mind, always look to tell a positive story. If you do decide to use a narrative that touches on something difficult, avoid going into unnecessary detail. You want to make sure that if you do choose to write about something difficult, focus on how you’ve overcome it, learned from it, and have a renewed sense of purpose as a result.

Proof-read and have others read it

This one is obvious, but if it isn’t, it should be. Taking just a few seconds to do a basic spell check can save you time, embarrassment and potentially, disappointment. You can also download extensions like Grammarly which make life even easier. Remember, while the law schools will read your LSAT essay, they appreciate that it is a quick essay, written under test pressure. The PS, on the other hand, is seen as something that is supposed to represent your very best writing, so missing something as simple as a spelling error, that could have been caught with a click of a button, isn’t a good look.

Finally, you need to have some of your friends or family give it a read, especially the ones who know you well, and can be honest with you. Does it make sense to them? Does it sound like you? What are their honest thoughts and feedback? Don’t get emotionally attached to the PS, if you get feedback, take it on board and adjust where needed.

Show, don’t tell

Finally, it is important that you don’t fall into the trap of ‘hard selling’ yourself. ‘I’ statements carry very little weight and can even come across as desperate – never a good look. For example, rather than saying ‘I am a good leader, I am responsible, I have the respect of my peers and I can overcome challenges,’ use a narrative that allows you to show those qualities. Remember, admissions committees review thousands of apps every year and your ‘hard sell’ is just you making claims about yourself. Instead, focus on telling a story that really leaves the reader with a clear sense of who you are, what makes you tick, and why you’d be a great fit at their school.

When I think of the personal statements I’ve read over the years, there are a few that really stand out. In each of those memorable essays, the story was allowed to speak for the author. Remember, while there are schools that will allow 3-4 double spaced pages for a PS (e.g. Baylor, UC Berkeley), most will state a limit of two, or will anticipate 2 pages if it isn’t stated. That isn’t a lot of real estate to work with, so you can’t tell three shallow stories in a single PS. Instead, choose one, and do a deep dive.

Economy of word usage is also key, just as it is in law, but allowing the story to speak for you will be not only more engaging for the reader (who is a decision maker) but it really allows you to make a meaningful impact. Never lose sight of the fact that a strong PS can move you from being waitlisted, or rejected, to being admitted.

One of my favorite essays was from an applicant who ultimately ended up at a top 10 law school. Instead of going on about their love of learning, strategy, humility, or desire to something difficult, this essay chose a narrative that left the reader with no doubt as to the fact that they had those qualities.

Without spelling out their essay, they took a pretty vanilla interaction they had, after trying a new hobby for the first time, and allowed that story to speak for them. It was powerful, without having to be dramatic, and resulted in them being admitted to their dream school.

Make no mistake, reflection, planning, and cold, clinical reviews can take an average PS and turn it into something that makes an admissions committee look for reasons to admit you.

The bottom line is this, the PS is a tricky bit of writing that asks you do to a lot, in an incredibly short essay. However, despite its difficulty, it is your chance to introduce yourself in a way that really allows you to make it clear what you bring to the table, and your ability as a written communicator. If done well, it can make all the difference when it comes to admission!

Posted: 7-7-2021

personal statement tips lsat

© 2022 Automatic Test Preparation

(844) ENGINE-1

The Magoosh logo is the word Magoosh spelled with each letter o replaced with a check mark in a circle.

How to Format Your Law School Personal Statement

Woman proofreading law school personal statement format - image by Magoosh

How do you write your law school personal statement? Well first of all, let’s make sure that we’re on the same page about what your personal statement is. Your personal statement is the one part of your law school application package and law school requirements that you have complete control over, so you’ll want to put your best foot forward. A personal statement will often focus on why you want to go to law school (or transfer law schools ), but it can also focus on a personal story or aspect of your life.

It shows what makes you unique and why a school should admit you. The personal statement should focus on you, your background, and your goals more broadly. Make sure that it adds something new to your application materials – the school already has your transcript, resume, etc. Think about what you really want the application committee to know about you.

Before anything else, a quick clarification: the law school personal statement is different from an optional essay , which can take on a variety of forms. This could include diversity statements, addendums, or other essays. Here are some examples of law school personal statements that may help you understand the task at hand better.

How do you format a law school personal statement?

In brief, here’s what your law school personal statement will need in terms of format:

  • Overall : No title, 11- or 12-point Times New Roman font, one-inch margins.
  • Header : Your name, your LSAC number, and “Personal Statement” with a page number, formatted as either one or three lines. Check with your school’s requirements.
  • Body : Double-spaced, left-aligned (or justified), paragraphs indented 0.5 inches and not separated with an extra line, single space after periods.
  • Ending : End as you would a normal essay. This isn’t a letter; no signature is needed.

Personal Statement Header

The header of the personal statement deserves a closer look. There are two ways of formatting this: either on one line, or on three. One line gives you more space on the page, but can look busy. Three lines have the opposite effect. Weigh the pros and cons based on the length of your statement, then format accordingly.

If you choose the one-line format, be sure to space your information out equally or separate it with punctuation (commas, dashes, and slashes work well) so that it reads clearly.

If you use a three-line format, separate information by line like this:

Name, Page Number LSAC # Personal Statement

Law School Personal Statement Format: FAQs

What should be included in a law school personal statement.

  • Who you are. Show readers that you’re an interesting person who brings experiences and skills that will benefit not only the campus community, but the larger legal community.You’re applying in a pool of thousands of candidates, so be sure to highlight what makes you stand out from your peers.  
  • Your true voice. There’s a reason why the personal statement isn’t just called a statement or an essay. Sometimes applicants feel that they should write pieces about public policy or social issues, but these too often fall short of showing an applicant’s true voice. Have someone you know well review your personal statement objectively. If they can’t tell you were the one who wrote it, it’s probably time for a rewrite .  
  • Specific information about that school. It’s not sufficient to say that you want to attend Santa Clara Law School for its good curriculum, strong faculty, and numerous clinic opportunities. Notice how you could replace “Santa Clara Law School” with any other law school’s name, and the sentence could still make sense? That tells Santa Clara admissions officers that you don’t know very much about their school. Which leads us to our next point…  
  • Research on the school itself. Figure out what makes the schools you’re applying to different from others. This is a great opportunity to reach out to alumni, and talk to the admissions staff! You can also use the Internet, visit your local bookstore and check out some guide books, or search around on online forums. Some schools are known for their strength in a certain area of law (think international law or intellectual property law ). Some schools are known for their commitment to pro bono work . Some schools’ faculty are renowned for their research in a specific discipline. Others offer distinctive programs or fellowships to their students. Identify what really interests you about the school, and tie that back to the academic and career interests you discuss in your personal statement.  
  • Reflections on the school’s environment. Perhaps you’re looking for a collegial law school environment that mirrors your own undergraduate experience at a small liberal arts school. Or perhaps you’re looking for a large law school so you can take advantage of the network and breadth of resources and alumni that a law school of that size can offer. And don’t forget about the environment outside the school building! Is it important that you have access to hiking trials? Or a ski slope to enjoy over winter break? Environment is often a key factor students consider when deciding on a particular law school, so don’t forget to mention it as a way to express your interest!  
  • Concise writing. Check your school’s website to determine how long your personal statement can be, and take it seriously. Law schools are not only looking at whether you can write concisely and effectively, but also whether you can follow posted instructions. Most schools only allow 2-5 pages for personal statement submissions. As a lawyer, you’ll need to write briefs and be able to clearly present client cases. Now’s the time to show that you are capable of honing your communication skills.  
  • Authenticity. Law schools aren’t asking you to establish your own NGO or be an Olympic athlete. Rather, they’re looking for candidates who help round out a class and contribute positively to their school. Plenty of people get admitted to law school each year who aren’t superhuman, so don’t feel a need to pretend you’re more accomplished than you are (or stretch the truth). Be yourself – and view this as part of helping the reader understand who you are.  
  • Correct writing. Maybe for class assignments, you’ve been able to submit the first draft you write as final. Or maybe one edit is typically sufficient for you to call an essay complete. For the law school personal statement, you want to commit at least two rounds of edits to perfecting your writing. Not only should you review your work, you should also ask both a friend and a fully objective reviewer (like a career center counselor or a campus writing tutor) to give feedback. Once you have at least two rounds of edits, read it out loud to yourself. This will help you identify any awkward phrases and typos. The more time you spend editing your writing, the more confident you’ll be in the strength of your personal statement.

What should you not write in a personal statement for law school?

  • Repetition . If your resume shows that you were vice president of your college’s botany club, general secretary of Basket-Weavers Anonymous, and founder of a campus-wide Pizza Appreciation Day, your personal statement need not repeat these things. Now, if founding Pizza Appreciation Day was such a transformative experience for you that you need to highlight it in your personal statement, be sure you’re telling admissions officers something new that your resume doesn’t already tell.  
  • Your autobiography . Admissions officers don’t need a play-by-play of your entire life’s events from day one. Autobiographies become long and rambling – two things your personal statement shouldn’t be. Focus on aspects of your life that truly differentiate you from others in a meaningful way.  
  • Academic issues . Law schools offer you space in a separate essay to explain academic discrepancies. Your personal statement is your chance to focus on the positive and show admissions officers you’d be an asset to their school. Don’t use your personal statement to go into detail about how your dog’s unexpected chronic migraines prevented you from getting a good GPA during your first year of college.  
  • Legal jargon . No, you’re not a lawyer yet – and law school admissions officers are not going to be impressed by legal jargon that’s used incorrectly or used as a way to show off. Keep your tone and language simple. Remember that your personal statement is meant to show your own voice.  
  • Cliches . Don’t be the student who bores admissions officers with another essay about how you want to be a lawyer because you like to argue. Avoid clichés – by definition, they’re overused and don’t add value. They make your personal statement generic, and you’ll fall flat when compared with other candidates.  
  • Other people . Your personal statement should keep the focus on you. It’s great if you want to write about how your famous lawyer uncle inspired you to join the legal profession, but make sure the essay remains true to your story – not your uncle’s.  
  • Slang . Admissions officers view the personal statement as a showcase of your best writing – so slang and casual English are best left behind. While you want your tone to be friendly, you don’t want to sound like you’re chatting with a best friend on a Friday night. Keep things professional.

How do I write a statement for law school?

There are three main steps to the writing process, and they’re no different here! Namely: brainstorm, write, and edit. In this case, though, we’ll add a fourth step: format and proofread.

  • Brainstorming is one of the most crucial things you can do for your personal statement. You want to make sure your ideas are strong, following the guidelines above. It can be helpful to spend a little quiet time alone or in a cozy coffee shop to start brainstorming. Check out some of our law school personal statement examples to spark ideas!  
  • Once you’ve brainstormed and organized your ideas, the writing itself will go pretty quickly. After you’ve written the first draft, leave the personal statement aside for a day or two (a week or more is better!). Then, come back. What parts don’t flow well? What ideas need more (or less) elaboration? Cut—and add—brutally! Editing is not the same as proofreading; this is the point at which you ensure the ideas themselves are sound.  
  • Now, it’s time for the final line edit-format-proofread. In a line edit, you’ll work to make sure you’re using the best possible words correctly, rephrasing and rewriting as needed. Then, use the law school personal statement format discussed above to organize the writing. Finally, read through for errors in spelling, grammar, and formatting.

Voila! Your law school personal statement is now ready. If you’re planning to send it off to a T14 law school, check out our post on the top law schools for more tips and information. And no matter what, check out our post on how to get into law school !

Need more help?

Improve your LSAT scores with Magoosh LSAT , you can choose between a live cohorted class with an instructor (which includes all our lessons and practice questions) or access to the self-study option by itself.

Rachel Kapelke-Dale

Rachel is one of Magoosh’s Content Creators. She writes and updates content on our High School and GRE Blogs to ensure students are equipped with the best information during their test prep journey. As a test-prep instructor for more than five years in there different countries, Rachel has helped students around the world prepare for various standardized tests, including the SAT, ACT, TOEFL, GRE, and GMAT, and she is one of the authors of our Magoosh ACT Prep Book . Rachel has a Bachelor of Arts in Comparative Literature from Brown University, an MA in Cinematography from the Université de Paris VII, and a Ph.D. in Film Studies from University College London. For over a decade, Rachel has honed her craft as a fiction and memoir writer and public speaker. Her novel, THE BALLERINAS , is forthcoming in December 2021 from St. Martin’s Press , while her memoir, GRADUATES IN WONDERLAND , co-written with Jessica Pan, was published in 2014 by Penguin Random House. Her work has appeared in over a dozen online and print publications, including Vanity Fair Hollywood. When she isn’t strategically stringing words together at Magoosh, you can find Rachel riding horses or with her nose in a book. Join her on Twitter , Instagram , or Facebook !

View all posts

More from Magoosh

Volunteer experience for law school personal statement examples - image by Magoosh

Leave a Reply Cancel reply

Your email address will not be published. Required fields are marked *

This site uses various technologies, as described in our Privacy Policy, for personalization, measuring website use/performance, and targeted advertising, which may include storing and sharing information about your site visit with third parties. By continuing to use this website you consent to our Privacy Policy and Terms of Use .

We are experiencing sporadically slow performance in our online tools, which you may notice when working in your dashboard. Our team is fully engaged and actively working to improve your online experience. If you are experiencing a connectivity issue, we recommend you try again in 10-15 minutes. We will update this space when the issue is resolved.

Law School Personal Statement Tips

In your personal statement for law school you want to present yourself as intelligent, professional, mature and persuasive. These are the qualities that make a good lawyer, so they're the qualities that law schools seek in applicants. Your grades and LSAT score are the most important part of your application to law school. But you shouldn't neglect the law school personal statement. Your application essay is a valuable opportunity to distinguish yourself from other applicants, especially those with similar LSAT scores and GPA.

law school personal statement

How To Write a Personal Statement for Law School

1. be specific to each law school ..

You'll probably need to write only one basic personal statement, but you should tweak it for each law school to which you apply. There are usually some subtle differences in what each school asks for in a personal statement.

2. Good writing is writing that is easily understood.

Good law students—and good lawyers—use clear, direct prose. Remove extraneous words and make sure that your points are clear. Don't make admissions officers struggle to figure out what you are trying to say.

Read More: Find Your Law School

3. Get plenty of feedback on your law school personal statement.

The more time you've spent writing your personal statement, the less likely you are to spot any errors. You should ask for feedback from professors, friends, parents, and anyone else whose judgment and writing skills you trust. This will help ensure that your statement is clear, concise, candid, structurally sound and grammatically accurate.

4. Find your unique angle.

Who are you? What makes you unique? Sometimes, law school applicants answer this question in a superficial way. It's not enough to tell the admissions committee that you're a straight-A student from Missouri. You need to give them a deeper sense of yourself. And there's usually no need to mention awards or honors you've won. That's what the law school application  or your resume is for.

Use your essay to explain how your upbringing, your education, and your personal and professional experiences have influenced you and led you to apply to law school. Give the admissions officers genuine insight into who you are. Don't use cliches or platitudes. The more personal and specific your personal statement is, the better received it will be.

Applying to law school? Use our  law school search to find the right program for you or browse our  law school ranking lists .

Practice for the LSAT

Take a LSAT practice test with us under the same conditions as the real thing. You'll get a personalized score report highlighting your strengths and areas of improvement.

START A FREE PRACTICE TEST

Featured Grad Schools For You

Explore Graduate Programs for You

Explore our featured graduate schools & programs to find those that both match your interests and are looking for students like you.

Best-Online-JD-Programs-2023

Best Online JD Programs

Online JD programs offer flexibility, affordability, access to innovative technologies, students from a diversity of career backgrounds, and global opportunities.

Best-Online-MSL-Programs

Best Online MSL Programs

Master of Studies in Law (MSL) programs are for non-lawyers whose careers in their organizations will benefit from legal training even though they are not required to be practicing attorneys.

Featured Business Schools For You

Find MBA Programs Matched to Your Interests

Explore our featured business schools to find those that are looking for students like you.

personal statement tips lsat

Free MCAT Practice Test

I already know my score.

personal statement tips lsat

MCAT Self-Paced 14-Day Free Trial

personal statement tips lsat

Enrollment Advisor

1-800-2REVIEW (800-273-8439) ext. 1

1-877-LEARN-30

Mon-Fri 9AM-10PM ET

Sat-Sun 9AM-8PM ET

Student Support

1-800-2REVIEW (800-273-8439) ext. 2

Mon-Fri 9AM-9PM ET

Sat-Sun 8:30AM-5PM ET

Partnerships

  • Teach or Tutor for Us

College Readiness

International

Advertising

Affiliate/Other

  • Enrollment Terms & Conditions
  • Accessibility
  • Cigna Medical Transparency in Coverage

Register Book

Local Offices: Mon-Fri 9AM-6PM

  • SAT Subject Tests

Academic Subjects

  • Social Studies

Find the Right College

  • College Rankings
  • College Advice
  • Applying to College
  • Financial Aid

School & District Partnerships

  • Professional Development
  • Advice Articles
  • Private Tutoring
  • Mobile Apps
  • International Offices
  • Work for Us
  • Affiliate Program
  • Partner with Us
  • Advertise with Us
  • International Partnerships
  • Our Guarantees
  • Accessibility – Canada

Privacy Policy | CA Privacy Notice | Do Not Sell or Share My Personal Information | Your Opt-Out Rights | Terms of Use | Site Map

©2024 TPR Education IP Holdings, LLC. All Rights Reserved. The Princeton Review is not affiliated with Princeton University

TPR Education, LLC (doing business as “The Princeton Review”) is controlled by Primavera Holdings Limited, a firm owned by Chinese nationals with a principal place of business in Hong Kong, China.

Blog | Blueprint Prep

Quick Tips for a Quick (But Effective) Personal Statement

M Hope Echales

  • January 9, 2019
  • Reviewed by: Matt Riley

personal statement tips lsat

In the greatest musical in recent history, Aaron Burr queries of Alexander Hamilton: “Why do you write like you’re running out of time?”

For those of you taking the January LSAT and hoping to apply to law school this application cycle , you should soon start writing like you’re running out of time. Because you are. As soon as possible following the examination, you should put quill to ink (i.e., fingers to keyboard) and get started on your personal statement . I recommend prioritizing the personal statement over other application related tasks because it involves the most mental labor — the rest of the process is largely busywork.

Without further ado, here are some quick tips for getting started on the personal statement:

1. Think about what makes you unique

The hardest part about the personal statement, by far, is coming up with a topic. In order to accomplish that first hurdle, you need to think about what makes your story unique and interesting. For some, that’s a life-changing experience abroad, for others it is recovering from a challenging situation (although preferably not a situation that reflects poorly on one’s judgment-making process), and for others it is an experience that opened their eyes to the importance of the law in some meaningful way. The key is to avoid writing something clichéd or non-unique.

For example, it turns out that a lot of aspiring lawyers end up doing Teach for America. Without casting any aspersions on TFA, my understanding is that it is an extraordinarily common subject for personal statements. The personal becomes impersonal if one’s experience is shared by hundreds of other applicants. I would avoid topics like this, unless you find a unique spin that you believe differentiates your experience from everyone else’s. Thus, the first step is to come up with some topics relating to you as an individual.

As soon as you settle on a topic, I would launch into writing. Get an initial draft done quickly (this is easier said than done). I was paralyzed with indecision for a long time before I started writing, and it slowed the process down substantially. The first draft doesn’t have to be perfect (or even good), but it needs to get done.

3. Get a little help from your friends

When I was writing my personal statement, I shared it with as many people as possible. Not only does this outsource some of the editing work, but it also allows you to get important input on the parts of your story that are compelling and the parts that are not. This is also why getting an initial draft done quickly is so important. You should refine and rework your statement numerous times before finalizing. It can be very difficult to share your personal statement, especially after the hours you’ll likely expend coming up with a topic and investing yourself in the drafting it, but it is a vital part of the process.

———— Writing the personal statement is draining and difficult. Selecting a topic, especially immediately after finishing the LSAT, is a difficult process. Just remember, apart from your grades and LSAT score, the personal statement is probably the most important part of your application. It is very important that you get the attention of the admission officer and display your writing aptitude.

If you need inspiration for the drafting process or feel like things couldn’t get worse, throw on the Hamilton soundtrack and think about trying to write dozens of Federalist Papers. Write like you’re running out of time.

Related Posts

applying to law school to do lis

A Step-By-Step Guide to Applying to Law School

  • July 23, 2024

Studying for the LSAT

What Is the LSAT? A Beginner’s Guide to the Law School Admission Test

  • June 27, 2024

personal statement tips lsat

How to Pay for Law School

  • May 20, 2024

Leave a Reply Cancel Reply

Your email address will not be published. Required fields are marked *

Name  *

Email  *

Add Comment  *

Save my name, email, and website in this browser for the next time I comment.

Post Comment

Login to your account

Remember Me

Register for a Free Account

Access sample lessons, a free LSAT PrepTest, and 100 question explanations today!

Password (twice) * password strength indicator

Analytics Identifier

Excellent Law School Personal Statement Examples By David Busis Published May 5, 2019 Updated Feb 10, 2021

We’ve rounded up five spectacular personal statements that helped students with borderline numbers get into T-14 schools. You’ll find these examples to be as various as a typical JD class. Some essays are about a challenge, some about the evolution of the author’s intellectual or professional journey, and some about the author’s identity. The only common thread is sincerity. The authors did not write toward an imagined idea of what an admissions officer might be looking for: they reckoned honestly with formative experiences.

Personal Statement about a Career Journey

The writer of this personal statement matriculated at Georgetown. Her GPA was below the school’s 25th percentile and her LSAT score was above the 75th percentile. She was not a URM.

* Note that we’ve used female pronouns throughout, though some of the authors are male.

I don’t remember anything being out of the ordinary before I fainted—just the familiar, heady feeling and then nothing. When I came to, they were wheeling me away to the ER. That was the last time I went to the hospital for my neurology observership. Not long after, I crossed “doctor” off my list of post-graduate career options. It would be best, I figured, if I did something for which the day-to-day responsibilities didn’t make me pass out.

Back at the drawing board, I reflected on my choices. The first time around, my primary concern was how I could stay in school for the longest amount of time possible. Key factors were left out of my decision: I had no interest in medicine, no aptitude for the natural sciences, and, as it quickly became apparent, no stomach for sick patients. The second time around, I was honest with myself: I had no idea what I wanted to do.

My college graduation speaker told us that the word “job” comes from the French word “gober,” meaning “to devour.” When I fell into digital advertising, I was expecting a slow and toothless nibbling, a consumption whose impact I could ignore while I figured out what I actually wanted to do. I’d barely started before I realized that my interviewers had been serious when they told me the position was sink or swim. At six months, I was one toothbrush short of living at our office. It was an unapologetic aquatic boot camp—and I liked it. I wanted to swim. The job was bringing out the best in me and pushing me to do things I didn’t think I could do.

I remember my first client emergency. I had a day to re-do a presentation that I’d been researching and putting together for weeks. I was panicked and sure that I’d be next on the chopping block. My only cogent thought was, “Oh my god. What am I going to do?” The answer was a three-part solution I know well now: a long night, lots of coffee, and laser-like focus on exactly and only what was needed.

Five years and numerous emergencies later, I’ve learned how to work: work under pressure, work when I’m tired, and work when I no longer want to. I have enough confidence to set my aims high and know I can execute on them. I’ve learned something about myself that I didn’t know when I graduated: I am capable.

The word “career” comes from the French word “carrière,” denoting a circular racecourse. Perhaps it shouldn’t surprise me then, that I’ve come full circle with regards to law school. For two college summers, I interned as a legal associate and wondered, “Is this for me?” I didn’t know if I was truly interested, and I was worried that even if I was, I wouldn’t be able to see it through. Today, I don’t have those fears.

In the course of my advertising career, I have worked with many lawyers to navigate the murky waters of digital media and user privacy. Whereas most of my co-workers went to great lengths to avoid our legal team, I sought them out. The legal conversations about our daily work intrigued me. How far could we go in negotiating our contracts to reflect changing definitions of an impression? What would happen if the US followed the EU and implemented wide-reaching data-protection laws?

Working on the ad tech side of the industry, I had the data to target even the most niche audiences: politically-active Mormon Democrats for a political client; young, low-income pregnant women for a state government; millennials with mental health concerns in a campaign for suicide prevention. The extent to which digital technology has evolved is astonishing. So is the fact that it has gone largely unregulated. That’s finally changing, and I believe the shift is going to open up a more prominent role for those who understand both digital technology and its laws. I hope to begin my next career at the intersection of those two worlds.

Personal Statement about Legal Internships

The writer of this essay was admitted to every T14 law school from Columbia on down and matriculated at a top JD program with a large merit scholarship. Her LSAT score was below the median and her GPA was above the median of each school that accepted her. She was not a URM.

About six weeks into my first legal internship, my office-mate gestured at the window—we were seventy stories high in the Chrysler Building—and said, with a sad smile, doesn’t this office just make you want to jump? The firm appeared to be falling apart. The managing partners were suing each other, morale was low, and my boss, in an effort to maintain his client base, had instructed me neither to give any information to nor take any orders from other attorneys. On my first day of work, coworkers warned me that the firm could be “competitive,” which seemed to me like a good thing. I considered myself a competitive person and enjoyed the feeling of victory. This, though, was the kind of competition in which everyone lost.

Although I felt discouraged about the legal field after this experience, I chose not to give up on the profession, and after reading a book that featured the U.S. attorney’s office for the Southern District of New York, I sent in an internship application. Shortly after, I received an offer to work at the office. For my first assignment, I attended a hearing in the federal courthouse. As I entered the magnificent twenty-third-floor courtroom, I felt the gravitas of the issue at hand: the sentencing of a terrorist.

That sense of gravitas never left me, and visiting the courtroom became my favorite part of the job. Sitting in hearings amidst the polished brass fixtures and mahogany walls, watching attorneys in refined suits prosecute terror, cybercrime, and corruption, I felt part of a grand endeavor. The spectacle enthralled me: a trial was like a combination of a theatrical performance and an athletic event. If I’d seen the dark side of competition at my first job, now I was seeing the bright side. I sat on the edge of my seat and watched to see if good—my side—triumphed over evil—the defense. Every conviction seemed like an unambiguous achievement. I told my friends that one day I wanted to help “lock up the bad guys.”

It wasn’t until I interned at the public defender’s office that I realized how much I’d oversimplified the world. In my very first week, I took the statement of a former high school classmate who had been charged with heroin possession. I did not know him well in high school, but we both recognized one another and made small talk before starting the formal interview. He had fallen into drug abuse and had been convicted of petty theft several months earlier. After finishing the interview, I wished him well.

The following week, in a courtroom that felt more like a macabre DMV than the hallowed halls I’d seen with the USAO, I watched my classmate submit his guilty plea, which would allow him to do community service in lieu of jail time. The judge accepted his plea and my classmate mumbled a quiet “thank you.” I felt none of the achievement I’d come to associate with guilty pleas. In that court, where hundreds of people trudged through endless paperwork and long lines before they could even see a judge, there were no good guys and bad guys—just people trying to put their lives back together.

A year after my internship at the public defender’s office, I read a profile of Preet Bharara, the U.S. attorney for the Southern District of New York, and my former boss. In the profile, he says, “You don’t want a justice system in which prosecutors are cowboys.” The more I saw at the public defender’s office, the more I rethought my experience at the USAO. When I had excitedly called my parents after an insider trading conviction, I had not thought of the defendant’s family. When I had cheered the conviction of a terrorist, I hadn’t thought about the fact that a conviction could not undo his actions. As I now plan on entering the legal profession—either as a prosecutor or public defender—I realize that my enthusiasm momentarily overwrote my empathy. I’d been playing cowboy. A lawyer’s job isn’t to lock up bad guys or help good guys in order to quench a competitive thirst—it’s to subsume his or her ego in the work and, by presenting one side of a case, create a necessary condition for justice.

Personal Statement about Cultural Identity

The writer of this essay was offered significant merit aid packages from Cornell, Michigan, and Northwestern, and matriculated at NYU Law. Her LSAT score was below the 25th percentile LSAT score and her GPA matched the median GPA of NYU.

By the age of five, I’d attended seven kindergartens and collected more frequent flier miles than most adults. I resided in two worlds – one with fast motorcycles, heavy pollution, and the smell of street food lingering in the air; the other with trimmed grass, faint traces of perfume mingling with coffee in the mall, and my mom pressing her hand against my window as she left for work. She was the only constant between these two worlds – flying me between Taiwan and America as she struggled to obtain a U.S. citizenship.

My family reunited for good around my sixth birthday, when we flew back to Taiwan to join my dad. I forgot about the West, acquired a taste for Tangyuan, and became fast friends with the kids in my neighborhood. In the evenings, I’d sit with my grandmother as she watched soap operas in Taiwanese, the dialect of the older generation, which I picked up in unharmonious bits and pieces. Other nights, she would turn off the TV, and speak to me about tradition and history – recounting my ancestors, life during the Japanese regime, raising my dad under martial law. “You are the last of the Li’s,” she would say, patting my back, and I’d feel a quick rush of pride, as though a lineage as deep as that of the English monarchy rested on my shoulders.

When I turned seven, my parents enrolled me in an American school, explaining that it was time for me, a Tai Wan Ren (Taiwanese), to learn English – “a language that could open doors to better opportunities.” Although I learned slowly, with a handful of the most remedial in ESL (English as a Second Language), books like The Secret Garden and The Wind in the Willows opened up new worlds of captivating images and beautiful stories that I longed to take part in.

Along with the new language, I adopted a different way to dress, new mannerisms, and new tastes, including American pop culture. I stopped seeing the neighborhood kids, and sought a set of friends who shared my affinity for HBO movies and  Claire’s Jewelry . Whenever taxi drivers or waitresses asked where I was from, noting that I spoke Chinese with too much of an accent to be native, I told them I was American.

At home, I asked my mom to stop packing Taiwanese food for my lunch. The cheap food stalls I once enjoyed now embarrassed me. Instead, I wanted instant mashed potatoes and Kraft mac and cheese.

When it came time for college, I enrolled in a liberal arts school on the East Coast to pursue my love of literature, and was surprised to find that my return to America did not feel like the full homecoming I’d expected. America was as familiar as it was foreign, and while I had mastered being “American” in Taiwan, being an American in America baffled me. The open atmosphere of my university, where ideas and feelings were exchanged freely, felt familiar and welcoming, but cultural references often escaped me. Unlike my friends who’d grown up in the States, I had never heard of Wonder Bread, or experienced the joy of Chipotle’s burrito bowls. Unlike them, I missed the sound of motorcycles whizzing by my window on quiet nights.

It was during this time of uncertainty that I found my place through literature, discovering Taiye Selasi, Edward Said, and Primo Levi, whose works about origin and personhood reshaped my conception of my own identity. Their usage of the language of otherness provided me with the vocabulary I had long sought, and revealed that I had too simplistic an understanding of who I was. In trying to discover my role in each cultural context, I’d confined myself within an easy dichotomy, where the East represented exotic foods and experiences, and the West, development and consumerism. By idealizing the latter and rejecting the former, I had reduced the richness of my worlds to caricatures. Where I am from, and who I am, is an amalgamation of my experiences and heritage: I am simultaneously a Mei Guo Ren and Taiwanese.

Just as I once reconciled my Eastern and Western identities, I now seek to reconcile my love of literature with my desire to effect tangible change. I first became interested in law on my study abroad program, when I visited the English courts as a tourist. As I watched the barristers deliver their statements, it occurred to me that law and literature have some similarities: both are a form of criticism that depends on close reading, the synthesis of disparate intellectual frameworks, and careful argumentation. Through my subsequent internships and my current job, I discovered that legal work possessed a tangibility I found lacking in literature. The lawyers I collaborate with work tirelessly to address the same problems and ideas I’ve explored only theoretically in my classes – those related to human rights, social contracts, and moral order. Though I understand that lawyers often work long hours, and that the work can be, at times, tedious, I’m drawn to the kind of research, analysis, and careful reading that the profession requires. I hope to harness my critical abilities to reach beyond the pages of the books I love and make meaningful change in the real world.

Personal Statement about Weightlifting

The writer of this essay was admitted to her top choice—a T14 school—with a handwritten note from the dean that praised her personal statement. Her LSAT score was below the school’s median and her GPA was above the school’s median.

As I knelt to tie balloons around the base of the white, wooden cross, I thought about the morning of my best friend’s accident: the initial numbness that overwhelmed my entire body; the hideous sound of my own small laugh when I called the other member of our trio and repeated the words “Mark died”; the panic attack I’d had driving home, resulting in enough tears that I had to pull off to the side of the road. Above all, I remembered the feeling of reality crashing into my previously sheltered life, the feeling that nothing was as safe or certain as I’d believed.

I had been with Mark the day before he passed, exactly one week before we were both set to move down to Tennessee to start our freshman year of college. It would have been difficult to feel so alone with my grief in any circumstance, but Mark’s crash seemed to ignite a chain reaction of loss. I had to leave Nashville abruptly in order to attend the funeral of my grandmother, who helped raise me, and at the end of the school year, a close friend who had helped me adjust to college was killed by an oncoming car on the day that he’d graduated. Just weeks before visiting Mark’s grave on his birthday, a childhood friend shot and killed himself in an abandoned parking lot on Christmas Eve. I spent Christmas Day trying to act as normally as possible, hiding the news in order not to ruin the holiday for the rest of my family.

This pattern of loss compounding loss affected me more than I ever thought it would. First, I just avoided social media out of fear that I’d see condolences for yet another friend who had passed too early. Eventually, I shut down emotionally and lost interest in the world—stopped attending social gatherings, stopped talking to anyone, and stopped going to many of my classes, as every day was a struggle to get out of bed. I hated the act that I had to put on in public, where I was always getting asked the same question —“I haven’t seen you in forever, where have you been?”—and always responding with the same lie: “I’ve just been really busy.”

I had been interested in bodybuilding since high school, but during this time, the lowest period of my life, it changed from a simple hobby to a necessity and, quite possibly, a lifesaver. The gym was the one place I could escape my own mind, where I could replace feelings of emptiness with the feeling of my heart pounding, lungs exploding, and blood flooding my muscles, where—with sweat pouring off my forehead and calloused palms clenched around cold steel—I could see clearly again.

Not only did my workouts provide me with an outlet for all of my suppressed emotion, but they also became the one aspect of my life where I felt I was still in control. I knew that if it was Monday, no matter what else was going on, I was going to be working out my legs, and I knew exactly what exercises I was going to do, and how many repetitions I was going to perform, and how much weight I was going to use for each repetition. I knew exactly when I would be eating and exactly how many grams of each food source I would ingest. I knew how many calories I would get from each of proteins, carbohydrates, and fats. My routine was one thing I could count on.

As I loaded more plates onto the barbell, I grew stronger mentally as well. The gym became a place, paradoxically, of both exertion and tranquility, a sanctuary where I felt capable of thinking about the people I’d lost. It was the healing I did there that let me tie the balloons to the cross on Mark’s third birthday after the crash, and that let me spend the rest of the afternoon sharing stories about Mark with friends on the side of the rural road. It was the healing I did there that left me ready to move on.

One of the fundamental principles of weightlifting involves progressively overloading the muscles by taking them to complete failure, coming back, and performing past the point where you last failed, consistently making small increases over time. The same principle helped me overcome my grief, and in the past few years, I’ve applied it to everything from learning Spanish to studying for the LSAT. As I prepare for the next stage of my life, I know I’ll encounter more challenges for which I’m unprepared, but I feel strong enough now to acknowledge my weaknesses, and—by making incremental gains—to overcome them.

Personal Statement about Sexual Assault

The writer of this essay was accepted to many top law schools and matriculated at Columbia. Her LSAT score matched Columbia’s median while her GPA was below Columbia’s 25th percentile.

My rapist didn’t hold a knife to my throat. My rapist didn’t jump out of a dark alleyway. My rapist didn’t slip me a roofie. My rapist was my eighth-grade boyfriend, who was already practicing with the high school football team. He assaulted me in his suburban house in New Jersey, while his mom cooked us dinner in the next room, in the back of an empty movie theatre, on the couch in my basement.

It started when I was thirteen and so excited to have my first real boyfriend. He was a football player from a different school who had a pierced ear and played the guitar. I, a shy, slightly chubby girl with a bad haircut and very few friends, felt wanted, needed, and possibly loved. The abuse—the verbal and physical harassment that eventually turned sexual—was just something that happened in grown-up relationships. This is what good girlfriends do, I thought. They say yes.

Never having had a sex-ed class in my life, it took me several months after my eighth-grade graduation and my entry into high school to realize the full extent of what he did to me. My overall experience of first “love” seemed surreal. This was something that happened in a Lifetime movie, not in a small town in New Jersey in his childhood twin bed. I didn’t tell anyone about what happened. I had a different life in a different school by then, and I wasn’t going to let my trauma define my existence.

As I grew older, I was confronted by the fact that rape is not a surreal misfortune or a Lifetime movie. It’s something that too many of my close friends have experienced. It’s when my sorority sister tells me about the upstairs of a frat house when she’s too drunk to say no. It’s when the boy in the room next door tells me about his uncle during freshman orientation. It’s a high school peer whose summer internship boss became too handsy. Rape is real. It’s happening every day, to mothers, brothers, sisters, and fathers—a silent majority that want to manage the burden on their own, afraid of judgement, afraid of repercussions, afraid of a he-said she-said court battle.

I am beyond tired of the silence. It took me three years to talk about what happened to me, to come clean to my peers and become a model of what it means to speak about something that society tells you not to speak about. Motivated by my own experience and my friends’ stories, I joined three groups that help educate my college community about sexual health and assault: New Feminists, Speak for Change, and Sexual Assault Responders. I trained to staff a peer-to-peer emergency hotline for survivors of sexual assault. I protested the university’s cover-up of a gang-rape in the basement of a fraternity house two doors from where I live now. As a member of my sorority’s executive board, I have talked extensively about safety and sexual assault, and have orchestrated a speaker on the subject to come to campus and talk to the exceptional young women I consider family. I’ve proposed a DOE policy change to make sexual violence education mandatory to my city councilman. This past summer, I traveled to a country notorious for sexual violence and helped lay the groundwork for a health center that will allow women to receive maternal care, mental health counseling, and career counseling.

Law school is going to help me take my advocacy to the next level. Survivors of sexual assault, especially young survivors, often don’t know where to turn. They don’t know their Title IX rights, they don’t know about the Clery Act, and they don’t know how to demand help when every other part of the system is shouting at them to be quiet and give up. Being a lawyer, first and foremost, is being an advocate. With a JD, I can work with groups like SurvJustice and the Rape Survivors Law Project to change the lives of people who were silenced for too long.

📌 Further reading:

  • Six Law School Personal Statements That Got Into Harvard
  • Free admissions course

All Categories

  • Admissions 200 Posts
  • Success Story 4 Posts
  • LSAT 231 Posts
  • Logical Reasoning 11 Posts
  • Logic Games 10 Posts
  • Reading Comprehension 5 Posts
  • Podcast 91 Posts
  • Uncategorized 35 Posts

Join our newsletter

Other posts.

With the incoming students arrived (hopefully in one piece!) and orientation complete (hopefully without incident!), law school admissions officers face one of their stranger time periods of the year. It’s […]

While last week’s blog detailed how law school admissions officers were preparing last-minute details for orientation, this is the week when there are no more “last minutes.” It’s the current […]

Listen and subscribe:Apple Podcasts | Spotify | Audible Join the 7Sage admissions consulting staff for a panel discussion on applying to top law schools! From supplemental statements, to interviews, to early decision, the panel […]

Leave a Reply Cancel

You must be logged in to post a comment. You can get a free account here .

Stetson Law Admissions Blog

  • Applying to Law School
  • Law School Insider Tips
  • Law School Prep
  • There are no suggestions because the search field is empty.

Personal Statement Tips from a Law School Student

Personal Statements are often thought of as the core of a law school application. Even if you have perfect grades and a perfect LSAT score, a poorly written personal statement will leave doubts about your ability to effectively communicate. Also, you may come across as arrogant because admissions will see you did not take a requirement seriously. This does not make you look good and might overshadow your high marks.

Conversely, a well written personal statement makes you look great and can give your application the boost it needs if your grades or LSAT score are less than perfect. A good personal statement can take you to so many places. Plus, it is great practice for writing cover letters to potential employers. You will write a few cover letters in law school. Yeah, you heard right…personal statements do not go away.

Here are my top three tips for writing your personal statement.

Friend helps her improve her personal statement for law school

1. Write down the traits you want to highlight about yourself.

Start with determining what you would like the admissions team to know about you. Highlight these traits early in your statement. Personal statements should only be one page, so you will want to make each sentence matter . Outlining what you want to tell admissions about yourself will keep your statement concise and focused. Also, reflecting on your best traits can build confidence during the application process.

2. Choose a story for your personal statement.

Stories. We all love them, and we all have them. Do not be afraid to tell yours. Our passions are largely based on our experiences. Tell admissions how and why an event or events influenced who you are today. Draw a clear connection between your experiences and their effect on your future goals. Stories can tell an admissions board so much about how you navigate the world. Regardless, if you chose to tell a story or not, be sure to express to admissions why they should select you and why you selected that law school.

3. Have someone else read your personal statement.

Have you ever thought something sounded good in your head but, when said, did not sound good out loud? Your writing is no different . We may think of something that sounds good to us but, when written down, confuses the reader. Without a reader other than yourself, you will continue to think it works and sounds good in your personal statement.

Do not let the admissions officer be the first human to read your personal statement. In fact, do not let admissions be the second or third person to read your personal statement. Have at least three other people read it before you send it in to admissions. When I applied, I had my mom, a law student, and an undergraduate professor read my personal statement. The feedback from all was helpful, especially because of their different perspectives. It showed me what did and didn’t work.

Now go and conquer that personal statement!

Molly Kaelin

Topics: Applying to Law School

Recommended Articles

Student Spotlight: Collin O'Keefe

Student Spotlight: Collin O'Keefe - Stetson J.D. Candidate '25

personal statement tips lsat

Student Spotlight: Tatum Carlson - Stetson J.D. Graduate '24

personal statement tips lsat

Student Spotlight: Rylie Pennell -         J.D. Candidate '24

Further reading.

Closeup of a person typing their personal statement on a laptop

Top 8 Most Common Mistakes When Writing a Personal Statement

personal statement tips lsat

What NOT To Say In Your Personal Statement

Woman working on her law school application on a laptop

How to Write an Addendum for Law School

Preparing for law school, let us help.

personal statement tips lsat

You might be using an unsupported or outdated browser. To get the best possible experience please use the latest version of Chrome, Firefox, Safari, or Microsoft Edge to view this website.

How To Prepare For The LSAT: Resources And Common Questions

Amy Boyington

Updated: Apr 17, 2024, 8:06am

How To Prepare For The LSAT: Resources And Common Questions

Law schools are known for their demanding admissions requirements. To apply to law school, aspiring law students typically write a law school personal statement , polish their résumés and send test scores to demonstrate their readiness to succeed in a rigorous curriculum.

The only standardized test accepted by every American Bar Association accredited law school is the Law School Admission Test (LSAT)®. The LSAT tests your reasoning, reading comprehension and writing skills as they apply to concepts covered in law school. Law schools use the LSAT to determine whether applicants are well-suited for a J.D. degree program. While the test is not a legal aptitude test, law schools rely upon it because of its ability to predict academic performance in the first year of law school.

Find out what to expect from the LSAT in this guide, which explores the test’s subject areas, its structure and how to prepare for the LSAT.

Why You Can Trust Forbes Advisor Education

Forbes Advisor’s education editors are committed to producing unbiased rankings and informative articles covering online colleges, tech bootcamps and career paths. Our ranking methodologies use data from the National Center for Education Statistics , education providers, and reputable educational and professional organizations. An advisory board of educators and other subject matter experts reviews and verifies our content to bring you trustworthy, up-to-date information. Advertisers do not influence our rankings or editorial content.

  • 6,290 accredited, nonprofit colleges and universities analyzed nationwide
  • 52 reputable tech bootcamp providers evaluated for our rankings
  • All content is fact-checked and updated on an annual basis
  • Rankings undergo five rounds of fact-checking
  • Only 7.12% of all colleges, universities and bootcamp providers we consider are awarded

What Is The LSAT?

The LSAT—offered by the Law School Admission Council (LSAC)®—is a standardized test for prospective law school students. The test examines your skills in reading, writing and reasoning, measuring your potential success in a law program. The LSAT features multiple-choice questions and a writing portion.

LSAT Questions

The multiple-choice section tests your critical thinking and reading comprehension skills through four 35-minute blocks of questions. The questions cover the following topics:

  • Reading comprehension. This section includes reading material typically found in law school. The questions gauge your ability to dissect information, draw conclusions and understand key points. The LSAT includes four reading passages, each with a series of follow-up questions.
  • Analytical reasoning. This section presents scenarios featuring facts, rules and relationships. You’ll answer questions using problem-solving skills to assemble the puzzle of each scenario while adhering to relevant rules or limitations. Starting in August 2024, this section will be eliminated, and instead the exam will include two sections dedicated to logical reasoning.
  • Logical reasoning. The LSAT’s logical reasoning portion includes source-based passages, each with up to two follow-up questions. The questions cover analogies, argument flaws and logical reasoning patterns.

The test also includes one unscored question section covering one of the above topic areas. LSAC uses these questions for quality assurance to strengthen the accuracy of future tests.

LSAT Writing®

LSAT Writing consists of a scenario prompt with two positions the writer can take. This section requires you to choose a side of an argument and use your reasoning and logic skills to support your choice in a persuasive essay. You complete this section separately from the rest of the LSAT, and you have 35 minutes to do so. LSAT Writing is monitored through live proctoring software, allowing test takers to complete it from their own homes, a development that came about, initially, in response to the Covid-19 pandemic.

LSAT Scoring

The LSAT counts each question you get right rather than subtracting questions you get wrong from a cumulative score. (In other words, there is no penalty for guessing.) LSAC does not score LSAT Writing samples.

The LSAT scoring scale ranges from 120 to 180. The final score report includes your score, your percentile rank compared to other test takers and a score band that highlights your proficiency in each area of the exam. LSAC sends your LSAT score report and writing sample to your law schools of choice.

Common Questions About the LSAT

How long does the lsat take.

The LSAT takes about three hours to finish, including all five sections and a break. You can take the 35-minute writing portion up to eight days before the day you take the multiple-choice portion of the LSAT.

The LSAT includes two 35-minute sections of multiple-choice questions, followed by a 10-minute break. The test then continues with its final two 35-minute multiple-choice sections.

You can request additional breaks between each section if you qualify for accommodations.

How Many Times Can You Take the LSAT?

You can take the LSAT up to seven times. However, you are limited to five tests per five-year reportable scoring period. These rules only apply to tests taken from September 2019 to the present. Therefore, any LSATs you took before that time don’t count toward your limit.

You can’t retake the LSAT if you receive a perfect score in the current or past five testing years.

Is the LSAT Required for Law School?

The LSAT isn’t required for all law schools. Some schools also accept the Graduate Record Examinations (GRE)® or the Graduate Management Admission Test (GMAT)™. Law schools that accept the GRE or GMAT also accept LSAT scores.

What Can You Expect on LSAT Test Day?

The LSAT is administered online with a remote proctor and in person at professional testing centers. You get a 10-minute break halfway through the test. You can leave the room during your break, but you may not use electronic devices and you must check in with your proctor before beginning the second half of the LSAT.

LSAC allows you to have earplugs, tissues, water in a transparent container and medication on your desk if needed. The test no longer allows scratch paper except as an approved accommodation, but the LSAT Writing interface includes a digital scratch paper section.

You can take the written portion before or after completing the standard LSAT. The LSAT Writing takes place online with a live proctor only.

What Is a Good LSAT Score?

LSAT scores range from 120 to 180, and the median LSAT score is about 152. However, each law school determines its minimum LSAT score for applicants, with more competitive schools preferring scores of 160 or higher.

Common Questions About LSAT Writing

Is lsat writing required for law school.

LSAT Writing is not required for law schools that do not require the LSAT for admission; you may instead take the GRE for these schools. However, if your school does require the LSAT, you’ll need to complete LSAT Writing. Your school will not consider your LSAT complete if it doesn’t include LSAT Writing.

Is LSAT Writing Scored?

No, LSAT Writing isn’t scored. LSAC sends your writing sample to the law schools to which you apply. Those schools evaluate your writing sample based on their own requirements.

Do Law Schools Actually Read Your Writing Sample?

Law schools that require LSAT scores read each applicant’s writing sample. A school can weigh the importance of LSAT Writing samples however they’d like when determining whether an applicant qualifies for admission.

Does LSAT Writing Have a Minimum or Maximum Word Count?

LSAT Writing does not set a minimum or maximum word count. Instead, test takers should focus on writing a well-planned, organized and thorough essay during the 35-minute testing portion.

Common Questions About LSAT Prep

When should you start preparing for the lsat.

Anyone preparing for the LSAT should study relevant content for at least several weeks. If you have a lot of time to study, you might feel ready after just one or two months of preparation. However, some test takers might need three months or more to feel better equipped for the LSAT.

What’s the Best LSAT Prep Strategy?

Preparing for the LSAT is an intensive process and systems like memorization that might aid in a college exam do not help when taking the LSAT. As a result, preparing for the LSAT is a process that normally requires months of study, rather than weeks. For example, a preparation cycle of three to six months is very common.

A good starting point is to simply take a full practice test, often called ‘a diagnostic’ so that you can establish a baseline score. In addition to working through a formalized curriculum it is important to remember that taking practice tests after completing that curriculum will form the backbone of your preparation and will also help you objectively assess your readiness to take the test formally.

Are There Free LSAT Prep Resources?

Yes, there are several free LSAT prep resources, including official LSAC LawHub practice tests. Khan Academy also provides a free online LSAT prep course. However, given the correlation between your LSAT score and the cost of attending law school, test takers are strongly encouraged to invest in their test preparation, although spending thousands of dollars is rarely necessary.

LSAT Prep Resources to Consider

Studying for the LSAT and practicing under test conditions should acclimate you to the test and the conditions you will encounter on test day, the idea being that it will help you counter test anxiety and properly understand this very difficult exam. Below are some of the most popular resources for LSAT preparation.

LSAC’s LawHub is free for test takers who want extra practice before taking the exam. The plan includes exam-like testing modules and four official practice tests with instant scoring to determine critical study areas. However, it is important to remember that the level of instruction is going to be very different from that experienced in commercial test preparation offerings.

LawHub Advantage

LawHub Advantage is a paid version of the LawHub free study program, priced at $115 annually. It includes about 60 practice exams, educational resources and a status tracker for law school applications. This is an excellent resource for someone who has learned how to understand the test and needs to practice the test under increasing amounts of time pressure, and under test conditions.

LSAT eBooks and Prep Books

LSAC provides several official eBooks and test prep books for purchase on Amazon and other online retailers. One of its most comprehensive books, “The New Official LSAT TriplePrep Volume 1™,” includes three recent practice tests with answer keys, a scoring conversion table and three LSAT Writing prompts.

Other books include a single or bundle of practice tests. Each book generally costs between $10 and $25, with lower-priced e-book versions available. However, experts note that learning the test through the use of a book alone can lead to learning bad habits that can cause plateaus and which can be difficult to break.

Khan Academy Official LSAT Prep

The Khan Academy Official LSAT Prep program is a free study resource that identifies which exam areas you may need to develop. It then uses that information to create a custom plan with lessons and skills practice. Khan Academy is an excellent starting place, but with law school tuition costing hundreds of thousands of dollars, even at average schools, investing in quality LSAT preparation is strongly encouraged.

The Cost of LSAT Prep

The cost of LSAT preparation programs varies widely and depends on the experience of the tutor, whether the content is provided in person (more costly) or online and the level of student to tutor interaction. For instance, some providers offer subscription fees under $100 but with minimal interaction. Others range from anywhere between $1,000- $5,500 but spending such a large sum is rarely necessary. Test takers will also be encouraged to note that many providers allow waivers for those with an LSAT approved fee waiver.

  • Best Online Criminal Justice Degree Programs
  • Best Master’s In Homeland Security Online
  • Best Master’s In Forensic Science Online Programs
  • Best Master’s In Criminology Online Programs
  • Best Ph.D. In Public Administration Online
  • Best Online Paralegal Degrees
  • Best Online Public Administration Degrees
  • How To Become A Criminal Investigator
  • How To Become A Lawyer
  • Careers In Law
  • What Can You Do With A Master Of Public Administration? 10 MPA Jobs To Know
  • Online Associate Degrees In Criminal Justice
  • Is Law School Worth It? How To Decide If Law School Is Right For You
  • Is Law School Worth It? 5 Factors To Consider
  • Tips For Law School Personal Statements: Examples, Resources And More
  • Paralegal Vs. Lawyer
  • The Top 13 Pre-Law Majors
  • 5 Types Of Law Degrees And Careers For Graduates
  • What Can You Do With A Law Degree? Careers, Salaries And Job Outlooks

Where To Earn An Online Criminal Justice Associate Degree In 2024

Where To Earn An Online Criminal Justice Associate Degree In 2024

Jessica Crosby, M. Ed.

Where To Earn A Ph.D. In Criminal Justice Online In 2024

Mikeie Reiland, MFA

Best Master’s In Criminology Online Programs Of 2024

Mariah St. John

Best Master’s In Homeland Security Online Of 2024

Genevieve Carlton Ph.D.

Best Ph.D. In Public Administration Online Of 2024

Garrett Andrews

Where To Earn An Online Forensic Psychology Master’s Degree In 2024

Suzie Glassman

As a self-proclaimed lifelong learner and former educator, Amy Boyington is passionate about researching and advocating for learners of all ages. For over a decade, Amy has specialized in writing parenting and higher education content that simplifies the process of comparing schools, programs and tuition rates for prospective students and their families. Her work has been featured on several online publications, including Online MBA, Reader’s Digest and BestColleges.

  • How I Got a 177
  • LSAT Logical Reasoning Course
  • LSAT Mastermind Group
  • 12-week (3 month)
  • 16-week (4 Month)
  • General Scheduling Tips
  • LSAT Articles
  • Logical Reasoning
  • Reading Comprehension
  • Logic Games
  • Law Schools
  • Personal Statement Editing
  • LSAT Logic Games Course
  • Join the LSAT Mastermind Group
  • 2 Month Plan
  • 10-week Plan
  • 3 Month Plan
  • 14-week Plan
  • 4 Month Plan
  • 5 Month Plan
  • 6 Month Plan

Recent Posts

What lsat score do you need to get into university of michigan, your guide to lsat fee waivers, 5 harsh truths that will make you better at the lsat.

  • Taking A “Cold” LSAT Diagnostic Exam
  • What Is A Good GPA For Law School?

LawSchooli

The Only 3 LSAT Writing Sample Tips You’ll Ever Need

I know the writing section isn’t scored and I’ve heard it doesn’t matter all that much, but I was wanting to get some overall suggestions on how to do it the way admissions people like. I’m taking the upcoming test and I haven’t given much thought to the writing portion. I’m confident in my writing skills so I would just appreciate any general advice. Thanks!

Thanks for the question! I think the best way to answer is to start by taking a look at a real writing sample prompt from a real previous LSAT. Here’s an example of a pretty typical writing sample (excerpted from the June 2007 LSAT ):

June 2007 Writing Sample Directions & Prompt

Directions: The scenario presented below describes two choices, either one of which can be supported on the basis of the information given. Your essay should consider both choices and argue for one over the other, based on the two specified criteria and the facts provided. There is no “right” or “wrong” choice: a reasonable argument can be made for either.

BLZ Stores, an established men’s clothing retailer with a chain of stores in a major metropolitan area, is selecting a plan for expansion. Using the facts below, write an essay in which you argue for one of the following plans over the other based on the following two criteria: • The company wants to increase its profits. • The company wants to ensure its long-term financial stability. The “national plan” is to open a large number of men’s clothing stores throughout the country over a short period of time. In doing this, the company would incur considerable debt. It would also have to greatly increase staff and develop national marketing and distribution capabilities. Many regional companies that adopted this strategy increased their profits dramatically. A greater number tried and failed, suffering severe financial consequences. BLZ is not well known outside its home area. Research indicates that the BLZ name is viewed positively by those who know it. National clothing chains can offer lower prices because of their greater buying power. BLZ currently faces increasingly heavy competition in its home region from such chains. The “regional plan” is to increase the number and size of stores in the company’s home region and upgrade their facilities, product quality, and service. This could be achieved for the most part with existing cash reserves. These upgrades would generally increase the prices that BLZ charges. In one trial store in which such changes were implemented, sales and profits have increased. The local population is growing. BLZ enjoys strong customer loyalty. Regional expansion could be accomplished primarily using BLZ’s experienced and loyal staff and would allow continued reliance on known and trusted suppliers, contractors, and other business connections.

So first off, I’ll tell you what I tell everyone about the writing sample: Don’t worry about it too much. As long as you take it seriously, give it your best effort, and demonstrate that you are capable of writing, in English, in a coherent manner, then the writing section of the LSAT is highly unlikely to help (or hurt) your application.

If in fact, someone actually reads your LSAT writing sample when reviewing your application, then they’re probably going to recognize that they are reading an ungraded essay that you wrote after an intense 3-hour period in which you took perhaps the most important exam of your life… and they’re probably going to read it (if at all) in light of that fact. I doubt that anyone has gone in there and written an exceptionally brilliant treatise so moving that it swayed an admissions decision.

On the other hand, if you blow it off entirely, or blatantly ignore the stimulus and write a diatribe against standardized testing, or something silly like that, well, that’s sort of thing might make an admissions committee question your character.

Ok, now having said that… here are a few tips on how to write a passable essay

Remember that “there is no “right” or “wrong” choice: a reasonable argument can be made for either.”

Don’t waste much time worrying about which side you argue for. The issue is designed in such a way that a reasonable argument can be made for either side. Read the prompt & pick whichever side you initially lean toward. Then focus on developing as strong an argument as possible for that side.

Be sure to follow the ‘rules’ given & stay on topic

Keep your argument on-topic! In the example above, we’re asked to argue for either the “ national ” or “ regional ” plan on the basis of two criteria: the company wants to  increase its profits  &  ensure its long-term financial stability . Stick to that task.

In reality, there are probably a million different paths that the company could take aside from the “ national”  and  “regional”  plans.

For example: don’t come up with and argue for an alternative  “acquisition”  plan in which the company buys other strong regional players, even if you think that is ultimately the best real-world answer. That’s not what the question asks of you. You’re asked to argue for the  “national”  plan OR the  “regional”  plan .   Do just that. S tick to the script.

In reality, there are also probably a million different criteria that the company could take into consideration aside from  “increase profits”  and  “ensure long-term financial stability.” 

For example: don’t come up with and develop an argument around an alternative  “environmental impact”  criteria that the company should take into consideration when making its decision. Even if you believe that is ultimately an important real-world consideration. That’s not what the question asks of you. You’re asked to weigh the given plans on the basis of the given criteria.  Do just that. Stick to the Script.

Consider organizing your response using a modified version of the “IRAC” methodology.

“ IRAC  (pronounced EYE-rack) is an acronym that stands for  Issue ,  Rule ,  Application , and  Conclusion . It functions as a methodology for legal analysis. The IRAC format is mostly used in hypothetical questions in law school and bar exams.” ( thanks, Wikipedia! )

Using IRAC is by no means required, so if you don’t find this tip useful, feel free to ignore it and write an otherwise well-organized essay. But you’re frequently going to be asked to argue using the IRAC method in law school… so using this general framework is a simple way to write a well-organized essay that will be familiar to anyone reviewing your law school application.

I ssue: state the issue that you are being asked to analyze.

R ule: state the rule (criterion)

A pplication: apply the criterion to the facts presented in each alternative choice

C onclusion: conclude that the position you’re arguing for is the better choice, given how the stated criterion apply to the facts.

' src=

University of Chicago, J.D., 2012 -- CLICK HERE to find out how I got a 177 on the LSAT . Ready to Kickstart your LSAT Prep? Join the LSAT Mastermind Study Group

Related Posts

personal statement tips lsat

First, thank you for all the great resources.

I am interested in joining the your Mastermind course, but I’m just at the very beginning of my study process (I’ve ordered books and looked through lots of online resources). Would you suggest starting the course now, or once I’ve gotten a better feel for each section of the test? I want to get the most out of it. Thank you again!

Best, Jackie

' src=

Hi Jackie –

It’s great to join at the beginning of your studies as you get access to all of the study schedules with the membership. Plus it is a lifetime membership, so you never have to pay for more time to stay active in the group. You can really join at any stage of prep. Some people join right before their test for the extra push, some have been active for over a year as they prepare. I joined at the beginning and found it extremely helpful.

Leave A Reply Cancel Reply

Save my name, email, and website in this browser for the next time I comment.

Type above and press Enter to search. Press Esc to cancel.

Sign In or Register

Welcome back.

Login to your account below.

"Interesting" Personal Statements (Ep. 713)

The LSAT Demon Team logo.

The guys help listener Wagma pick a personal statement topic.

Get the Reddit app

The Reddit LSAT Forum. The best place on Reddit for LSAT advice. The Law School Admission Test (LSAT) is the test required to get into an ABA law school. Check out the sidebar for intro guides. Post any questions you have, there are lots of redditors with LSAT knowledge waiting to help.

Tips on personal statement?

Y’all 😩 anyone care to offer any pointers on how to begin my ps?? I hate writing about myself and I just keep staring at the blank page. What’s a good way to get the flow going? Thx!

By continuing, you agree to our User Agreement and acknowledge that you understand the Privacy Policy .

Enter the 6-digit code from your authenticator app

You’ve set up two-factor authentication for this account.

Enter a 6-digit backup code

Create your username and password.

Reddit is anonymous, so your username is what you’ll go by here. Choose wisely—because once you get a name, you can’t change it.

Reset your password

Enter your email address or username and we’ll send you a link to reset your password

Check your inbox

An email with a link to reset your password was sent to the email address associated with your account

Choose a Reddit account to continue

IMAGES

  1. personal statement outline

    personal statement tips lsat

  2. How To Write An Lsat Addendum Essay

    personal statement tips lsat

  3. What’s With the LSAT Writing Sample

    personal statement tips lsat

  4. Proofread my Essay: Lsat personal statement examples

    personal statement tips lsat

  5. 5 Key Tips For Writing a Great Personal Statement

    personal statement tips lsat

  6. A Crash Course for the LSAT Personal Statement

    personal statement tips lsat

VIDEO

  1. LSAT guessing strategy: always pick C!?

  2. Personal Statement-stravaganza (Ep. 309)

  3. How to write a Personal Statement?

  4. Mackenzie's Personal Statement Review

  5. LSAT Preparation to Achieve Your Highest LSAT Score

  6. LSAT Wrong Answer Journal & Law School Personal Statement

COMMENTS

  1. Law School Personal Statement: The Ultimate Guide (Examples Included)

    Part 1: Introduction. The law school admissions process can feel confusing, scary, and overwhelming. Questions like "What LSAT score do I need?", "How many law schools should I apply to?," and "Do law school rankings matter?" likely weigh on your mind.. But amid all the uncertainty, there's one thing we know for sure: the two most important components of your law school ...

  2. How to Write a Law School Personal Statement

    Personal Statement Body Section. The body of your personal statement should focus on the details of your story. Each paragraph should expand on your points and begin with a topic sentence that expresses the main idea of the paragraph in which it occurs. Ending sentences for body paragraphs should wrap up your points and help transition the ...

  3. How Do I Write My Personal Statement?

    Ask your mentor who's in law school to read it. Take feedback with a grain of salt but ask each reader to tell you what this essay shows them about who you are. Listen carefully to each response, and if you notice a pattern, take it seriously. The personal statement is the most important essay you will write in your law school admissions journey.

  4. The Ultimate Law School Personal Statement Resource List

    In the midst of the application season, one of the most common questions we receive is about crafting personal statements. Over the years, we've assembled an impressive array of free resources for students. If you're still struggling with finalizing your statement, you may find that one of the following tools helps you put everything in place.

  5. Law School Personal Statement

    The perfect personal statement is the one that perfectly and truly states you. If you bring your personality, your voice, and your story with honesty and authenticity, you'll have the start of a winning essay. ... Tips to Help You Ace the LSAT. LSATMax Claims 2017 Best LSAT Review Course Award. Plan Your Studying. Categories.

  6. Guide to Writing an Outstanding Law School Personal Statement · LSData

    Be precise and concise. Legal writing is known for its clarity and brevity, so practice these skills in your personal statement. Aim to keep it between 500 and 700 words, as brevity is the soul of wit (and law school applications). 5. Revision: The Art of Legal Editing.

  7. Law School Personal Statement Examples

    The personal statement is a critical part of a law school application. Although it may not be as important as your LSAT score and GPA, remember that law school admissions committees are trying to build a diverse and interesting community of students. Your personal statement is the primary way you can show law schools who you are beyond your numbers and resume, and is also an opportunity to ...

  8. Tips For Law School Personal Statements: Examples, Resources ...

    A law school personal statement is a multi-paragraph essay or narrative highlighting the reason you are pursuing a J.D. degree. This essay is an opportunity to share your identity with an ...

  9. Writing Your Personal Statement Part 1: Take Your Time

    Make it personal and individual. Treat it like an interview with the Admissions Committee. Keep it interesting. Use it as an insight into who you are. Be clear, concise, and organized. If you can write interesting, personal, compelling, clear, and effective personal statement in 24 hours or less, hats off to you.

  10. A Sample Structure for Your Law School Personal Statement

    The basic formula: I am. I did. I do. I will. Ben and I talk endlessly about showing, rather than telling, any time a personal statement comes up on the Thinking LSAT Podcast. The point is to demonstrate your strengths and achievements via facts, rather than forcing conclusions down the reader's throat.

  11. 5 Key Tips For Writing a Great Personal Statement

    Your personal statement is meant to be about you, not about your best friend, or your sister, or even how you think the world works. Of course you will include some discussion of the world around you and the people in your life to make your story clear and meaningful, but you should be writing much more about yourself than about anything else.

  12. 9 Important Personal Statement Tips for Law School Applicants

    Tip 3: Be genuine. You don't need to be a superhero to impress the law school admissions committee. You can show your passion, dedication, and law school readiness in lots of everyday anecdotes from your life. You can even write your personal statement about a mistake or a weakness—just make sure you turn it around to show how you ...

  13. Personal statements aren't easy, but here's how to write ...

    Don't get emotionally attached to the PS, if you get feedback, take it on board and adjust where needed. Show, don't tell. Finally, it is important that you don't fall into the trap of 'hard selling' yourself. 'I' statements carry very little weight and can even come across as desperate - never a good look.

  14. How to Format Your Law School Personal Statement

    Header: Your name, your LSAC number, and "Personal Statement" with a page number, formatted as either one or three lines. Check with your school's requirements. Body: Double-spaced, left-aligned (or justified), paragraphs indented 0.5 inches and not separated with an extra line, single space after periods.

  15. Law School Personal Statement Tips

    There are usually some subtle differences in what each school asks for in a personal statement. 2. Good writing is writing that is easily understood. Good law students—and good lawyers—use clear, direct prose. Remove extraneous words and make sure that your points are clear. Don't make admissions officers struggle to figure out what you are ...

  16. Quick Tips for a Quick (But Effective) Personal Statement

    I recommend prioritizing the personal statement over other application related tasks because it involves the most mental labor — the rest of the process is largely busywork. Without further ado, here are some quick tips for getting started on the personal statement: 1. Think about what makes you unique. The hardest part about the personal ...

  17. Excellent Law School Personal Statement Examples

    Excellent Law School Personal Statement Examples. By David Busis Published May 5, 2019 Updated Feb 10, 2021. We've rounded up five spectacular personal statements that helped students with borderline numbers get into T-14 schools. You'll find these examples to be as various as a typical JD class. Some essays are about a challenge, some ...

  18. Personal Statement Tips from a Law School Student

    A good personal statement can take you to so many places. Plus, it is great practice for writing cover letters to potential employers. You will write a few cover letters in law school. Yeah, you heard right…personal statements do not go away. Here are my top three tips for writing your personal statement. 1.

  19. How To Prepare For The LSAT: Resources And Common Questions

    As a result, preparing for the LSAT is a process that normally requires months of study, rather than weeks. For example, a preparation cycle of three to six months is very common. A good starting ...

  20. Quick Tips: Crafting a Top-Notch Personal Statement

    Apart from your LSAT and undergraduate GPA, the personal statement is certainly the most important aspect of your law school application. It's the one part of your file that is solely you.It's your voice, your experiences, your story, told in your words.Every day, as we help students put together exceptional applications as part of our law school admissions consulting programs.

  21. The Only 3 LSAT Writing Sample Tips You'll Ever Need

    pplication: apply the criterion to the facts presented in each alternative choice. onclusion: conclude that the position you're arguing for is the better choice, given how the stated criterion apply to the facts. admisssions LSAT Soft Factors writing writing sample. University of Chicago, J.D., 2012 -- CLICK HERE to find out how I got a 177 ...

  22. "Interesting" Personal Statements (Ep. 713)

    Admissions 101 Writing Sample Tips LSAT and Law School Rankings Not Going Away. Podcasts. ... "Interesting" Personal Statements (Ep. 713) LSAT Demon Team. The guys help listener Wagma pick a personal statement topic. The LSAT; Logic Games; Logical Reasoning; Reading Comprehension; Test Strategies; What is LSAT Demon? Sample LSAT Questions; 2024 ...

  23. Tips on personal statement? : r/LSAT

    The Reddit LSAT Forum. The best place on Reddit for LSAT advice. The Law School Admission Test (LSAT) is the test required to get into an ABA law school. Check out the sidebar for intro guides. Post any questions you have, there are lots of redditors with LSAT knowledge waiting to help.